Does the fact that we can only measure the two-way speed of light undermine the axiom of invariance?What if we shine a laser to east and another to west, will they arrive at the same time?Can we measure the speed of light in one direction?How can we show that the speed of light is really constant in all reference frames?Why is light speed the only constant speed?How did Einstein know the speed of light was constant?How can I show that the speed of light in vacuum is the same in all reference frames?Why can't we measure the one-way speed of light?

Is the tap water in France safe to drink?

Grade changes with auto grader

Nobel prize in literature 2018 - what is "encyclopedic passion"?

Prisoner's dilemma formulation for children

Stare long enough and you will have found the answer

How to deal with people whose priority is to not get blamed?

What sport was she watching?

Could an American state survive nuclear war?

Is it allowed to let the engine of an aircraft idle without a pilot in the plane. (For both helicopters and aeroplanes)

TikZ, forest and star macro

If you revoke a certificate authority's certificate, do all of the certificates it issued become invalid as well?

How to increment the value of a (decimal) variable (with leading zero) by +1?

Tube Patch or tubeless plug

What's the meaning of Electrical Inches?

When was the famous "sudo warning" introduced? Under what background? By whom?

How can I communicate feelings to players without impacting their agency?

When I am told that a dipole must have a determined length, does that include the space between both elements?

In this day and age should the definition / categorisation of erotica be revised?

Does any politician - honestly - want a No Deal Brexit?

How can a stock trade for a fraction of a cent?

What is the "5th Edition Adventures" book series?

Why is こんばんみ used as a response to こんばんは?

Why do some audio amps use AC while others use DC

rasterio "invalid dtype: 'bool'"



Does the fact that we can only measure the two-way speed of light undermine the axiom of invariance?


What if we shine a laser to east and another to west, will they arrive at the same time?Can we measure the speed of light in one direction?How can we show that the speed of light is really constant in all reference frames?Why is light speed the only constant speed?How did Einstein know the speed of light was constant?How can I show that the speed of light in vacuum is the same in all reference frames?Why can't we measure the one-way speed of light?






.everyoneloves__top-leaderboard:empty,.everyoneloves__mid-leaderboard:empty,.everyoneloves__bot-mid-leaderboard:empty
margin-bottom:0;









11














$begingroup$


When we measure the speed of light we get the same answer in all directions. This is taken to undermine the aether or absolute motion hypothesis and give support to the proposal that the speed of light is invariant, from which derives the theory of special relativity.



But doesn't the fact that we only measure speed of light 'there and back' undermine this conclusion? Wouldn't we expect this result through an aether?










share|cite|improve this question












$endgroup$










  • 2




    $begingroup$
    Hi @Andrew, you said “Wouldn't we expect this result through an aether?”. Actually, no, if you work out the math the two way speed of light is not isotropic in a standard aether theory.
    $endgroup$
    – Dale
    May 19 at 2:49











  • $begingroup$
    Why do you think we can only measure "there and back"? For instance, one of the first measurements of the speed of light was made by noting variations in the timing of eclipses of Jupiter's moons, which would seem to involve measuring only the "back" reflection of the sun's light: google.com/…
    $endgroup$
    – jamesqf
    May 19 at 5:58










  • $begingroup$
    @jamesqf Wikipedia says that although Rømer's measurement of Jupiter's moons seems to give a one-way speed, careful analysis shows that those calculations assume that the travel speed is isotropic. However, I must confess I don't fully understand the arguments given on that page.
    $endgroup$
    – PM 2Ring
    May 19 at 10:03






  • 2




    $begingroup$
    Careful analysis shows that Rømer's measurement says nothing about the one-way speed of light. physics.stackexchange.com/questions/456243/…
    $endgroup$
    – Albert
    May 19 at 10:43

















11














$begingroup$


When we measure the speed of light we get the same answer in all directions. This is taken to undermine the aether or absolute motion hypothesis and give support to the proposal that the speed of light is invariant, from which derives the theory of special relativity.



But doesn't the fact that we only measure speed of light 'there and back' undermine this conclusion? Wouldn't we expect this result through an aether?










share|cite|improve this question












$endgroup$










  • 2




    $begingroup$
    Hi @Andrew, you said “Wouldn't we expect this result through an aether?”. Actually, no, if you work out the math the two way speed of light is not isotropic in a standard aether theory.
    $endgroup$
    – Dale
    May 19 at 2:49











  • $begingroup$
    Why do you think we can only measure "there and back"? For instance, one of the first measurements of the speed of light was made by noting variations in the timing of eclipses of Jupiter's moons, which would seem to involve measuring only the "back" reflection of the sun's light: google.com/…
    $endgroup$
    – jamesqf
    May 19 at 5:58










  • $begingroup$
    @jamesqf Wikipedia says that although Rømer's measurement of Jupiter's moons seems to give a one-way speed, careful analysis shows that those calculations assume that the travel speed is isotropic. However, I must confess I don't fully understand the arguments given on that page.
    $endgroup$
    – PM 2Ring
    May 19 at 10:03






  • 2




    $begingroup$
    Careful analysis shows that Rømer's measurement says nothing about the one-way speed of light. physics.stackexchange.com/questions/456243/…
    $endgroup$
    – Albert
    May 19 at 10:43













11












11








11


2



$begingroup$


When we measure the speed of light we get the same answer in all directions. This is taken to undermine the aether or absolute motion hypothesis and give support to the proposal that the speed of light is invariant, from which derives the theory of special relativity.



But doesn't the fact that we only measure speed of light 'there and back' undermine this conclusion? Wouldn't we expect this result through an aether?










share|cite|improve this question












$endgroup$




When we measure the speed of light we get the same answer in all directions. This is taken to undermine the aether or absolute motion hypothesis and give support to the proposal that the speed of light is invariant, from which derives the theory of special relativity.



But doesn't the fact that we only measure speed of light 'there and back' undermine this conclusion? Wouldn't we expect this result through an aether?







special-relativity speed-of-light






share|cite|improve this question
















share|cite|improve this question













share|cite|improve this question




share|cite|improve this question








edited May 18 at 23:48









Qmechanic

115k13 gold badges229 silver badges1364 bronze badges




115k13 gold badges229 silver badges1364 bronze badges










asked May 18 at 20:24









AndrewAndrew

776 bronze badges




776 bronze badges










  • 2




    $begingroup$
    Hi @Andrew, you said “Wouldn't we expect this result through an aether?”. Actually, no, if you work out the math the two way speed of light is not isotropic in a standard aether theory.
    $endgroup$
    – Dale
    May 19 at 2:49











  • $begingroup$
    Why do you think we can only measure "there and back"? For instance, one of the first measurements of the speed of light was made by noting variations in the timing of eclipses of Jupiter's moons, which would seem to involve measuring only the "back" reflection of the sun's light: google.com/…
    $endgroup$
    – jamesqf
    May 19 at 5:58










  • $begingroup$
    @jamesqf Wikipedia says that although Rømer's measurement of Jupiter's moons seems to give a one-way speed, careful analysis shows that those calculations assume that the travel speed is isotropic. However, I must confess I don't fully understand the arguments given on that page.
    $endgroup$
    – PM 2Ring
    May 19 at 10:03






  • 2




    $begingroup$
    Careful analysis shows that Rømer's measurement says nothing about the one-way speed of light. physics.stackexchange.com/questions/456243/…
    $endgroup$
    – Albert
    May 19 at 10:43












  • 2




    $begingroup$
    Hi @Andrew, you said “Wouldn't we expect this result through an aether?”. Actually, no, if you work out the math the two way speed of light is not isotropic in a standard aether theory.
    $endgroup$
    – Dale
    May 19 at 2:49











  • $begingroup$
    Why do you think we can only measure "there and back"? For instance, one of the first measurements of the speed of light was made by noting variations in the timing of eclipses of Jupiter's moons, which would seem to involve measuring only the "back" reflection of the sun's light: google.com/…
    $endgroup$
    – jamesqf
    May 19 at 5:58










  • $begingroup$
    @jamesqf Wikipedia says that although Rømer's measurement of Jupiter's moons seems to give a one-way speed, careful analysis shows that those calculations assume that the travel speed is isotropic. However, I must confess I don't fully understand the arguments given on that page.
    $endgroup$
    – PM 2Ring
    May 19 at 10:03






  • 2




    $begingroup$
    Careful analysis shows that Rømer's measurement says nothing about the one-way speed of light. physics.stackexchange.com/questions/456243/…
    $endgroup$
    – Albert
    May 19 at 10:43







2




2




$begingroup$
Hi @Andrew, you said “Wouldn't we expect this result through an aether?”. Actually, no, if you work out the math the two way speed of light is not isotropic in a standard aether theory.
$endgroup$
– Dale
May 19 at 2:49





$begingroup$
Hi @Andrew, you said “Wouldn't we expect this result through an aether?”. Actually, no, if you work out the math the two way speed of light is not isotropic in a standard aether theory.
$endgroup$
– Dale
May 19 at 2:49













$begingroup$
Why do you think we can only measure "there and back"? For instance, one of the first measurements of the speed of light was made by noting variations in the timing of eclipses of Jupiter's moons, which would seem to involve measuring only the "back" reflection of the sun's light: google.com/…
$endgroup$
– jamesqf
May 19 at 5:58




$begingroup$
Why do you think we can only measure "there and back"? For instance, one of the first measurements of the speed of light was made by noting variations in the timing of eclipses of Jupiter's moons, which would seem to involve measuring only the "back" reflection of the sun's light: google.com/…
$endgroup$
– jamesqf
May 19 at 5:58












$begingroup$
@jamesqf Wikipedia says that although Rømer's measurement of Jupiter's moons seems to give a one-way speed, careful analysis shows that those calculations assume that the travel speed is isotropic. However, I must confess I don't fully understand the arguments given on that page.
$endgroup$
– PM 2Ring
May 19 at 10:03




$begingroup$
@jamesqf Wikipedia says that although Rømer's measurement of Jupiter's moons seems to give a one-way speed, careful analysis shows that those calculations assume that the travel speed is isotropic. However, I must confess I don't fully understand the arguments given on that page.
$endgroup$
– PM 2Ring
May 19 at 10:03




2




2




$begingroup$
Careful analysis shows that Rømer's measurement says nothing about the one-way speed of light. physics.stackexchange.com/questions/456243/…
$endgroup$
– Albert
May 19 at 10:43




$begingroup$
Careful analysis shows that Rømer's measurement says nothing about the one-way speed of light. physics.stackexchange.com/questions/456243/…
$endgroup$
– Albert
May 19 at 10:43










6 Answers
6






active

oldest

votes


















13
















$begingroup$

The bottom line is that this is the wrong question to ask. You don't ever prove an axiom in physics.



You're not quite right about the ether: while the first order effect cancels out in "there and back again" experiments, the second order effect doesn't, which is why the Michelson-Morley experiment stood a chance of testing for the ether. But Michelson-Morley wasn't the end of ether theory, because you could always add fixes to account for the results. For example, the famous Lorentz contraction in special relativity was originally invented for ether theory; the idea was that flowing ether physically squeezed objects smaller. Einstein just took effects like these more seriously.



Ether theory limped on for another 40 years, getting progressively more complicated as more results came in. Ether was not abandoned because it was disproven by experiments or because special relativity was proven by experiments, because this never happens. It was abandoned by the 1930s it could only explain experiments using tons of epicycles, while special relativity just worked perfectly out of the box.






share|cite|improve this answer










$endgroup$










  • 3




    $begingroup$
    To be fair, you don't ever prove an axiom. Thus is the nature of axioms.
    $endgroup$
    – tox123
    May 19 at 23:58






  • 4




    $begingroup$
    While it's true that you don't ever prove an axiom in the mathematical sense, you do attempt to falsify them in science. If, after many attempts to falsify it, you have failed to disprove it, then you may accept it as true. This is known as inductive reasoning, and it would typically be regarded as "proving" the theory in the common, non-mathematical vernacular. (Not to diminish the importance of understanding the difference, but we ought make sure we account for these things when dealing with misunderstandings.)
    $endgroup$
    – jpmc26
    May 20 at 5:45











  • $begingroup$
    @jpmc26 The point is that ether isn’t falsified either. You can almost always add tweaks to avoid falsification, so using that alone is not good enough.
    $endgroup$
    – knzhou
    May 20 at 10:57










  • $begingroup$
    For example, you could even take an extremely weak formulation of the ether which is just identical to GR but picks out some frame as special, without any physical consequences. That is not falsified any more than GR is. We don’t use it because it complicates things for no reason.
    $endgroup$
    – knzhou
    May 20 at 10:59


















13
















$begingroup$

There-and-back measurements still show the effect of an aether, because you can compare the results in different directions. For example, there-and-back along the direction of motion would show a different speed from there-and-back across it. This was the approach taken in the Michelson-Morley experiment.






share|cite|improve this answer










$endgroup$














  • $begingroup$
    Comments are not for extended discussion; this conversation has been moved to chat.
    $endgroup$
    – Chris
    May 19 at 16:54


















4
















$begingroup$

That the one-way speed of light is not observable plays a large role in keeping various relativistically-correct aether theories alive. Like general relativity, these new aether theories locally reduce to special relativity in the absence of nearby massive objects. Unlike general relativity, these new aether theories have a preferred universal frame of reference, typically a frame co-moving with the cosmic microwave background radiation.






share|cite|improve this answer










$endgroup$










  • 1




    $begingroup$
    If such theories cannot be ruled out, then why shouldn't they be amongst the respectable opinions that thinkers and researchers may choose to adopt and work with?
    $endgroup$
    – Steve
    May 19 at 0:40






  • 1




    $begingroup$
    @Steve “Choose to adopt and work with” is not a binary. People make different choices. Lots of people find GR useful; few find The Flying Spaghetti God hypothesis useful. Other theories are in between in popularity, i.e the MOND family. Nor is this decision time-invariant: theories and hypotheses are more and less popular as people show them more or less useful.
    $endgroup$
    – Bob Jacobsen
    May 19 at 1:32






  • 2




    $begingroup$
    @BobJacobsen, useful for what purpose? Perhaps when working with aether concepts attracts comparisons to the Flying Spaghetti God, you make it dis-useful for professional scientists to adopt such views and perform such work? And my point is to rebut the tone in which David speaks of apparently tenable theories being "kept alive" - the writer's implication being that these theories ought to be dead. Like I say, if they cannot be ruled out, then they are fair game for further scientific enquiry.
    $endgroup$
    – Steve
    May 19 at 1:53






  • 3




    $begingroup$
    @Steve, general relativity is a simple theory. Its only arbitrary parameter is Newton's gravitational constant. Other viable theories carry additional arbitrary parameters. In the limit that these parameters approach some value, typically zero, these theories become equivalent to general relativity. To date, all observations are consistent with these free parameters taking on the value that makes the alternative theory equivalent to general relativity.
    $endgroup$
    – David Hammen
    May 19 at 2:51






  • 1




    $begingroup$
    @DavidHammen, that's basically what I said above in my original comment to Bob's answer, that as it stands today (and as it stood in about 1920) Lorentz's aether theory (in its latter form) is equivalent to relativity - they are not mutually exclusive theories. It is only the general philosophical approach and concepts they employ which differ.
    $endgroup$
    – Steve
    May 19 at 9:13


















0
















$begingroup$

The $x$ direction is not any different physically than the $-x$ direction. So light travelling towards $x-> infty$ should travel in the same way it travels towards $x-> -infty$ wether theres an aether or not.
What should make a difference is if we measure the speed of light in the direction the body that emitted the light is moving through the aether and any other direction, for example, the direction perpendicular to this movement to see the difference a bit easier. However no difference in arrival times was measured.






share|cite|improve this answer










$endgroup$






















    0
















    $begingroup$

    This is not really intended to answer the question explicitly; rather to introduce a point in the traditional argument about a 'luminiferous aether', and more particularly the role of 'Michelson-Morley'-type experiments in repudiating the idea of such a medium for light propagation, which appears to be universally overlooked: namely, that since what was cast into considerable doubt, and effectively disproved by the original M-M experiment, was only the proposition that the Earth at all may move relatively to such an 'aether', one entirely plausible possibility nevertheless remains for such an observation--that nothing at all may be conceived to move relative to such an 'aether', in which case it may be concluded that the 'aether' is in fact unitary, universal and exclusive, and therefore equivalent to the fact of a unitary universal substance or fabric of which all of realty is comprised.



    Permitting this as a premise, what is immediately clear then is that since we too are composed of such a substance, it is not possible ever in any objective sense to ascertain its ultimate nature; and we are forever constrained solely to an understanding of its dynamic principles, an understanding to which we are admitted by virtue of the principle of resonance which must inhere within a universal unity: we are able continually to perceive the operation of its singular dynamic through this faculty of resonance defining consciousness itself.



    In the pursuit of such understanding however, we must first ascribe to such a unitary substance some correspondingly singular and exclusive property, 'a priori' or on the logical grounds of innate reason, which if it is chosen correctly represents the de facto equivalent of the substance itself, the knowledge of whose ultimate nature we are denied; and in this regard, we are perhaps indebted to ancient clarity--the single conceivable property of a universal unity is that it 'holds together' or coheres, according to what Aristotle and his forbears called a property of 'hexis' (compounded with 'entelos', effectively a 'direction', as of process), and which we are entitled since the days of Galileo to call a 'force', namely a 'cohesive force' not merely permeating reality but effectively constituting the entirety of it. No other means of apprehending that entirety or any other part of it from a given aspect exists except through the perception or inference of the action of such a force (and all the principal equations of physics further attest to this).



    Now since such a 'cohesive force' operating exclusively and throughout reality from every imaginable spatial point must further be imagined to operate from within space itself, thus from within a dimension of 'interior spatial depth' proceeding within any such locus--and no doubt M-theory and Kaluza-Klein theory before it alludes mathematically to such a dimension (noting that it is necessary in the conception of such a dimension only to extrapolate any 3rd dimension of Euclidean space interiorly provided that the properties of distribution of 'cohesive force' themselves are permitted to imply the multiplicity of 'space-time dimensions' in string theory)--, then it is necessary in order to account for the state of reality, that it does not for instance collapse upon itself at any such locus under the influence of this 'interior suction', not only that a perpetual disparity exists between the components of this 'cohesive force'--effectively constituting this dynamic universal substance (or re-imagined 'aether')--, but to infer a principle of equilibration between such loci laterally which amounts to its distribution, in linear components, over planar areas, and within volumes.



    It is this interplay, inevitably oscillatory, between the force operating 'into spatial depth' at any point, and its tendency to universal distribution laterally between loci under the impetus of an equilibrating principle implicit in a dynamic of cohesion or coherence which may be argued to imply not only the wave characteristics of the universal substance itself, of which the light wave is the primary and what may be imagined optimally regular manifestation, but a finite wave speed which implies both the finite speed of light and the basis of Einstein's second postulate of the invariance of c. The universal substance is therefore not to be construed as a medium for light propagation in the old-fashioned manner of an 'aether'; rather the light wave speed is the wave speed of that substance itself.



    Considering the basis of that wave dynamic intrinsic to what amounts to a 'unitary universal cohesive field' (comprising 'cohesive loci' defined by resultants or vectors), there is a limit to what may be said here; suffice to establish that an interaction between disparate components of 'cohesive force' or 'cohesive loci'* implies a resultant amounting to the elaboration of a further locus which is not only displaced in the direction of the stronger component, but because further interactions are implied between resultant loci, an oscillatory effect ensues with respect to an initial reference position whose progressive aspect due to that displacement further implies tendencies to recurrence of a given resultant with respect to that reference: viz. a wave effect is elaborated whose components or wave vectors are exclusively resultants in 'cohesive force'.



    However, since the fundamental wave dynamic and by inference light itself is more fundamentally a 'wave interference effect' between an interior spatial and effectively Euclidean spatial component, there arises not only a primary progressive effect implied by the disparity between these two components ostensible in the Euclidean spatial axis, but a further regularly recurring residual component implied with respect to an initial reference point in that axis by the properties of progression and recurrence of that primary dynamic. That is, in the case of the unitary wave principle imagined, that dynamic of recurrence is characterised by a residual component which is itself progressive and therefore definitive of a further level of resolution within this dynamic of wave recurrence, and it is this component which corresponds to and defines the wavelengths of EMR and in its condition of optimal regularity, the wavelengths of visible light. It is also this incrementally progressive aspect of the unitary wave interference principle in its various facets which endows the unitary cohesive space with its 'quantum nature' or properties. *(These linear resultants in 'cohesive force' may loosely be characterised as 'loci' by virtue of the predominance of the interior spatial component; moreover, all loci are effectively aspects of a singular universal context, thus of each other).



    With respect to the second postulate then, since within a singular substance characterised by the operation of a unitary 'cohesive force' in disparate components and by a universal wave principle, all motion may be understood eventually as wave motion, including that of a rigid body, the inertial fame moving with a velocity v and a light source attached to it in Einstein's scenario, and given that the elaboration of light--and the effect of our 'perceptual resonance' with it--is inherent within such a universal substance, then if it is supposed that within this singular medium light speed c represents the maximum ostensible wave speed intrinsic to that substance, as in the vacuum* (as one appreciates from Maxwell's equations), no manner of interference between or aggregation of wave velocities may exceed that speed c. The combination of a moving inertial frame, a light source and a light wave propagating from it constitutes a continuous wave effect with maximal wave speed. *(That specific maximum arises by virtue of the effect that wave progression implicit within such a medium in that limiting instance is innately imbued with a property of optimum regularity; corresponding to the principle of stationary action).



    Moreover, if it is further proposed that light itself derives more fundamentally in an interference effect between two wave components or vectors--or components of 'cohesive force'--, one projecting from the interior spatial dimension according to a particular geometry in which that dimension is represented in the pole of a cube (whose vertices correspond to interacting 'cohesive loci'), and one proceeding progressively in the Euclidean spatial dimension represented by the diagonal of the face of such a cube in which that first component projects, such that the familiar electromagnetic properties of light may be inferred from that interaction, then some conceptual latitude exists, effectively within the rotational aspect of such an interference effect in 'cubic cohesive space', for the accommodation of the velocity of a light source within the maximum allowable wave velocity of the universal medium.



    That is, at the origin of the light source--in whatever context this is imagined--, those components of a progressive recurrent wave dynamic constituting the velocity ascribed to its material substance imagined in the direction of its motion are effectively incorporated into the effect of propagating light itself in that direction, presumably in the form of its energy: viz. the velocity of a light source is effectively zero in the direction in which light wave speed is considered (for a light source moving at c must be indistinguishable from the propagating light itself). One may for example be required to imagine that a conservation of the L orbital angular momentum of electrons in the source as these yield kinetic energy to propagating photons implies an effective loss of linear velocity in the direction of motion of the source equivalent to that velocity of the source.



    I might add that since the discussion generally regarding the physical explanation for Einstein's second postulate appears ordinarily to conclude that no fundamental reason has yet been established for the evident veracity of this postulate or what is therefore effectively an axiom, one feels justified in indulging in creative speculation on the issue. Additionally, as one explores such a speculative geometric model of a unitary universal substance and its principle of universal resonance, explanations for certain otherwise inexplicable observations in physics, in QED in particular, become readily apparent; any and all of which are contingent upon the assumption of that unitary substance based on the premise of what is argued as a self-evident or axiomatic truth ('analytic a priori' after Kant): that of the inviolate universal unity of reality.






    share|cite|improve this answer










    $endgroup$






















      -1
















      $begingroup$

      Round - trip measurement of speed of light says nothing about the one - way speed of light. Effects of Lorentz contraction of moving bodies (MM experiment) and dilation of moving clock (Kennedy Thorndike experiment) had been first introduced in the framework of Lorentz aether theory.



      Since the same mathematical formalism occurs in both, it is not possible to distinguish between Lorentz aether theory and special relativity by experiment.



      The one-way speed of light is apparently anisotropic, the Sagnac effect (page 42 in this paper) is the best evidence of that.



      Remember, that in rotating frames, even in special relativity, the non-transitivity of Einstein synchronization diminishes its usefulness. If clock 1 and clock 2 are not synchronized directly, but by using a chain of intermediate clocks, the synchronization depends on the path chosen. Synchronization around the circumference of a rotating disk gives a non-vanishing time difference that depends on the direction used.



      Hence, the one – way speed of light relatively to the Earth surface is anisotropic, and it is very unlikely that one way speed of light is c an all relatively moving inertial frames.



      On the simplest example of the floating in a water ships this paper simulates all kinematic effects of special relativity (length contraction, time dilation, relativistic velocity addition, relativistic and transverse Doppler effects, Twin paradox, Bell‘s spaceship paradox, symmetry of observations). Things that may seem quirky and unusual take very simple shape, as soon as "absolute" time and medium had been introduced.



      The chapter Transverse Doppler Effect makes clear, that „conditionally moving“ clock ticks slower (transverse redshift) than „conditionally resting“ one , while „conditionally resting“ tick faster (transverse blueshift) than the "moving one". It is clear, that different synchronization procedures (standard isotropic for "stationary") frame and (non-standard anisotropic for "moving" frame) is needed to obtain the same result conducting measurements with the pair of synchronized clocks.



      The article on special relativity in Wikipedia teaches, that: "The Principle of Invariant Light Speed – "... light is always propagated in empty space with a definite velocity [speed] c which is independent of the state of motion of the emitting body" (from the preface).[p 1] That is, light in vacuum propagates with the speed c (a fixed constant, independent of direction) in at least one system of inertial coordinates (the "stationary system"), regardless of the state of motion of the light source."



      Apparently, according to the article, there is at least one "special stationary frame" where speed of light is isotropic and some others, where the speed of light is anisotropic.






      share|cite|improve this answer












      $endgroup$










      • 2




        $begingroup$
        The Sagnac effect is most emphatically not evidence for light speed anisotropy. The Sagnac effect is predicted by special relativity which assumes isotropy. To claim that it demonstrates anisotropy is only evidence that you do not know your facts enough to produce a credible answer! -1
        $endgroup$
        – Dale
        May 19 at 10:49






      • 1




        $begingroup$
        Especially for YOU I have posted the links to the Sagnac effect explanation in Gron's paper (it is written black on white there, that in rotating frame speed of light is anisotropic ) and the link to Einstein synchronization in Wikipedia. Could you please be so kind to clean your glasses and read it again? Or maybe you think that the Earth does not rotate? These views are out to date, I am afraid to say.
        $endgroup$
        – Albert
        May 19 at 11:00







      • 1




        $begingroup$
        That the speed of light is anisotropic in a rotating reference frame does not imply that it is evidence for light speed anisotropy. For something to be evidence of a hypothesis it must not only be predicted by a hypothesis but also it must be not predicted by the alternative hypothesis. The Sagnac effect is equally predicted both a non-isotropic and an isotropic hypothesis. Therefore it is not evidence of either
        $endgroup$
        – Dale
        May 19 at 11:10







      • 1




        $begingroup$
        Another minus one. The Sagnac effect is not evidence that the one-way speed of light is anisotropic in inertial frames. It is evidence that the one-way speed of light is anisotropic in rotating frames, which of course are not inertial. The Sagnac effect is fully consistent with special relativity. If this observed fact was inconsistent with special relativity, we wouldn't be teaching special relativity any more. It would instead be yet another idea tossed on the "oops, that turned out to be wrong and useless" pile of failed theories, just like the many failed theories of heat.
        $endgroup$
        – David Hammen
        May 19 at 15:24







      • 1




        $begingroup$
        @Dale, agreed, excuse me on that point. I think when I wrote I attributed Albert's apparent mis-paraphrasing of you, to your own actual words. As long as we are all now agreed that there is no conclusive evidence either way, and the one-way speed may or may not be constant, then that is the state of current science on the question. Special relativity is a theory that treats the one-way speed as constant (without establishing the fact by any experiment), other theories are tenable which do not treat the one-way speed as constant.
        $endgroup$
        – Steve
        May 19 at 19:22













      Your Answer








      StackExchange.ready(function()
      var channelOptions =
      tags: "".split(" "),
      id: "151"
      ;
      initTagRenderer("".split(" "), "".split(" "), channelOptions);

      StackExchange.using("externalEditor", function()
      // Have to fire editor after snippets, if snippets enabled
      if (StackExchange.settings.snippets.snippetsEnabled)
      StackExchange.using("snippets", function()
      createEditor();
      );

      else
      createEditor();

      );

      function createEditor()
      StackExchange.prepareEditor(
      heartbeatType: 'answer',
      autoActivateHeartbeat: false,
      convertImagesToLinks: false,
      noModals: true,
      showLowRepImageUploadWarning: true,
      reputationToPostImages: null,
      bindNavPrevention: true,
      postfix: "",
      imageUploader:
      brandingHtml: "Powered by u003ca class="icon-imgur-white" href="https://imgur.com/"u003eu003c/au003e",
      contentPolicyHtml: "User contributions licensed under u003ca href="https://creativecommons.org/licenses/by-sa/4.0/"u003ecc by-sa 4.0 with attribution requiredu003c/au003e u003ca href="https://stackoverflow.com/legal/content-policy"u003e(content policy)u003c/au003e",
      allowUrls: true
      ,
      noCode: true, onDemand: true,
      discardSelector: ".discard-answer"
      ,immediatelyShowMarkdownHelp:true
      );



      );














      draft saved

      draft discarded
















      StackExchange.ready(
      function ()
      StackExchange.openid.initPostLogin('.new-post-login', 'https%3a%2f%2fphysics.stackexchange.com%2fquestions%2f480938%2fdoes-the-fact-that-we-can-only-measure-the-two-way-speed-of-light-undermine-the%23new-answer', 'question_page');

      );

      Post as a guest















      Required, but never shown


























      6 Answers
      6






      active

      oldest

      votes








      6 Answers
      6






      active

      oldest

      votes









      active

      oldest

      votes






      active

      oldest

      votes









      13
















      $begingroup$

      The bottom line is that this is the wrong question to ask. You don't ever prove an axiom in physics.



      You're not quite right about the ether: while the first order effect cancels out in "there and back again" experiments, the second order effect doesn't, which is why the Michelson-Morley experiment stood a chance of testing for the ether. But Michelson-Morley wasn't the end of ether theory, because you could always add fixes to account for the results. For example, the famous Lorentz contraction in special relativity was originally invented for ether theory; the idea was that flowing ether physically squeezed objects smaller. Einstein just took effects like these more seriously.



      Ether theory limped on for another 40 years, getting progressively more complicated as more results came in. Ether was not abandoned because it was disproven by experiments or because special relativity was proven by experiments, because this never happens. It was abandoned by the 1930s it could only explain experiments using tons of epicycles, while special relativity just worked perfectly out of the box.






      share|cite|improve this answer










      $endgroup$










      • 3




        $begingroup$
        To be fair, you don't ever prove an axiom. Thus is the nature of axioms.
        $endgroup$
        – tox123
        May 19 at 23:58






      • 4




        $begingroup$
        While it's true that you don't ever prove an axiom in the mathematical sense, you do attempt to falsify them in science. If, after many attempts to falsify it, you have failed to disprove it, then you may accept it as true. This is known as inductive reasoning, and it would typically be regarded as "proving" the theory in the common, non-mathematical vernacular. (Not to diminish the importance of understanding the difference, but we ought make sure we account for these things when dealing with misunderstandings.)
        $endgroup$
        – jpmc26
        May 20 at 5:45











      • $begingroup$
        @jpmc26 The point is that ether isn’t falsified either. You can almost always add tweaks to avoid falsification, so using that alone is not good enough.
        $endgroup$
        – knzhou
        May 20 at 10:57










      • $begingroup$
        For example, you could even take an extremely weak formulation of the ether which is just identical to GR but picks out some frame as special, without any physical consequences. That is not falsified any more than GR is. We don’t use it because it complicates things for no reason.
        $endgroup$
        – knzhou
        May 20 at 10:59















      13
















      $begingroup$

      The bottom line is that this is the wrong question to ask. You don't ever prove an axiom in physics.



      You're not quite right about the ether: while the first order effect cancels out in "there and back again" experiments, the second order effect doesn't, which is why the Michelson-Morley experiment stood a chance of testing for the ether. But Michelson-Morley wasn't the end of ether theory, because you could always add fixes to account for the results. For example, the famous Lorentz contraction in special relativity was originally invented for ether theory; the idea was that flowing ether physically squeezed objects smaller. Einstein just took effects like these more seriously.



      Ether theory limped on for another 40 years, getting progressively more complicated as more results came in. Ether was not abandoned because it was disproven by experiments or because special relativity was proven by experiments, because this never happens. It was abandoned by the 1930s it could only explain experiments using tons of epicycles, while special relativity just worked perfectly out of the box.






      share|cite|improve this answer










      $endgroup$










      • 3




        $begingroup$
        To be fair, you don't ever prove an axiom. Thus is the nature of axioms.
        $endgroup$
        – tox123
        May 19 at 23:58






      • 4




        $begingroup$
        While it's true that you don't ever prove an axiom in the mathematical sense, you do attempt to falsify them in science. If, after many attempts to falsify it, you have failed to disprove it, then you may accept it as true. This is known as inductive reasoning, and it would typically be regarded as "proving" the theory in the common, non-mathematical vernacular. (Not to diminish the importance of understanding the difference, but we ought make sure we account for these things when dealing with misunderstandings.)
        $endgroup$
        – jpmc26
        May 20 at 5:45











      • $begingroup$
        @jpmc26 The point is that ether isn’t falsified either. You can almost always add tweaks to avoid falsification, so using that alone is not good enough.
        $endgroup$
        – knzhou
        May 20 at 10:57










      • $begingroup$
        For example, you could even take an extremely weak formulation of the ether which is just identical to GR but picks out some frame as special, without any physical consequences. That is not falsified any more than GR is. We don’t use it because it complicates things for no reason.
        $endgroup$
        – knzhou
        May 20 at 10:59













      13














      13










      13







      $begingroup$

      The bottom line is that this is the wrong question to ask. You don't ever prove an axiom in physics.



      You're not quite right about the ether: while the first order effect cancels out in "there and back again" experiments, the second order effect doesn't, which is why the Michelson-Morley experiment stood a chance of testing for the ether. But Michelson-Morley wasn't the end of ether theory, because you could always add fixes to account for the results. For example, the famous Lorentz contraction in special relativity was originally invented for ether theory; the idea was that flowing ether physically squeezed objects smaller. Einstein just took effects like these more seriously.



      Ether theory limped on for another 40 years, getting progressively more complicated as more results came in. Ether was not abandoned because it was disproven by experiments or because special relativity was proven by experiments, because this never happens. It was abandoned by the 1930s it could only explain experiments using tons of epicycles, while special relativity just worked perfectly out of the box.






      share|cite|improve this answer










      $endgroup$



      The bottom line is that this is the wrong question to ask. You don't ever prove an axiom in physics.



      You're not quite right about the ether: while the first order effect cancels out in "there and back again" experiments, the second order effect doesn't, which is why the Michelson-Morley experiment stood a chance of testing for the ether. But Michelson-Morley wasn't the end of ether theory, because you could always add fixes to account for the results. For example, the famous Lorentz contraction in special relativity was originally invented for ether theory; the idea was that flowing ether physically squeezed objects smaller. Einstein just took effects like these more seriously.



      Ether theory limped on for another 40 years, getting progressively more complicated as more results came in. Ether was not abandoned because it was disproven by experiments or because special relativity was proven by experiments, because this never happens. It was abandoned by the 1930s it could only explain experiments using tons of epicycles, while special relativity just worked perfectly out of the box.







      share|cite|improve this answer













      share|cite|improve this answer




      share|cite|improve this answer










      answered May 19 at 12:54









      knzhouknzhou

      56.8k14 gold badges161 silver badges275 bronze badges




      56.8k14 gold badges161 silver badges275 bronze badges










      • 3




        $begingroup$
        To be fair, you don't ever prove an axiom. Thus is the nature of axioms.
        $endgroup$
        – tox123
        May 19 at 23:58






      • 4




        $begingroup$
        While it's true that you don't ever prove an axiom in the mathematical sense, you do attempt to falsify them in science. If, after many attempts to falsify it, you have failed to disprove it, then you may accept it as true. This is known as inductive reasoning, and it would typically be regarded as "proving" the theory in the common, non-mathematical vernacular. (Not to diminish the importance of understanding the difference, but we ought make sure we account for these things when dealing with misunderstandings.)
        $endgroup$
        – jpmc26
        May 20 at 5:45











      • $begingroup$
        @jpmc26 The point is that ether isn’t falsified either. You can almost always add tweaks to avoid falsification, so using that alone is not good enough.
        $endgroup$
        – knzhou
        May 20 at 10:57










      • $begingroup$
        For example, you could even take an extremely weak formulation of the ether which is just identical to GR but picks out some frame as special, without any physical consequences. That is not falsified any more than GR is. We don’t use it because it complicates things for no reason.
        $endgroup$
        – knzhou
        May 20 at 10:59












      • 3




        $begingroup$
        To be fair, you don't ever prove an axiom. Thus is the nature of axioms.
        $endgroup$
        – tox123
        May 19 at 23:58






      • 4




        $begingroup$
        While it's true that you don't ever prove an axiom in the mathematical sense, you do attempt to falsify them in science. If, after many attempts to falsify it, you have failed to disprove it, then you may accept it as true. This is known as inductive reasoning, and it would typically be regarded as "proving" the theory in the common, non-mathematical vernacular. (Not to diminish the importance of understanding the difference, but we ought make sure we account for these things when dealing with misunderstandings.)
        $endgroup$
        – jpmc26
        May 20 at 5:45











      • $begingroup$
        @jpmc26 The point is that ether isn’t falsified either. You can almost always add tweaks to avoid falsification, so using that alone is not good enough.
        $endgroup$
        – knzhou
        May 20 at 10:57










      • $begingroup$
        For example, you could even take an extremely weak formulation of the ether which is just identical to GR but picks out some frame as special, without any physical consequences. That is not falsified any more than GR is. We don’t use it because it complicates things for no reason.
        $endgroup$
        – knzhou
        May 20 at 10:59







      3




      3




      $begingroup$
      To be fair, you don't ever prove an axiom. Thus is the nature of axioms.
      $endgroup$
      – tox123
      May 19 at 23:58




      $begingroup$
      To be fair, you don't ever prove an axiom. Thus is the nature of axioms.
      $endgroup$
      – tox123
      May 19 at 23:58




      4




      4




      $begingroup$
      While it's true that you don't ever prove an axiom in the mathematical sense, you do attempt to falsify them in science. If, after many attempts to falsify it, you have failed to disprove it, then you may accept it as true. This is known as inductive reasoning, and it would typically be regarded as "proving" the theory in the common, non-mathematical vernacular. (Not to diminish the importance of understanding the difference, but we ought make sure we account for these things when dealing with misunderstandings.)
      $endgroup$
      – jpmc26
      May 20 at 5:45





      $begingroup$
      While it's true that you don't ever prove an axiom in the mathematical sense, you do attempt to falsify them in science. If, after many attempts to falsify it, you have failed to disprove it, then you may accept it as true. This is known as inductive reasoning, and it would typically be regarded as "proving" the theory in the common, non-mathematical vernacular. (Not to diminish the importance of understanding the difference, but we ought make sure we account for these things when dealing with misunderstandings.)
      $endgroup$
      – jpmc26
      May 20 at 5:45













      $begingroup$
      @jpmc26 The point is that ether isn’t falsified either. You can almost always add tweaks to avoid falsification, so using that alone is not good enough.
      $endgroup$
      – knzhou
      May 20 at 10:57




      $begingroup$
      @jpmc26 The point is that ether isn’t falsified either. You can almost always add tweaks to avoid falsification, so using that alone is not good enough.
      $endgroup$
      – knzhou
      May 20 at 10:57












      $begingroup$
      For example, you could even take an extremely weak formulation of the ether which is just identical to GR but picks out some frame as special, without any physical consequences. That is not falsified any more than GR is. We don’t use it because it complicates things for no reason.
      $endgroup$
      – knzhou
      May 20 at 10:59




      $begingroup$
      For example, you could even take an extremely weak formulation of the ether which is just identical to GR but picks out some frame as special, without any physical consequences. That is not falsified any more than GR is. We don’t use it because it complicates things for no reason.
      $endgroup$
      – knzhou
      May 20 at 10:59













      13
















      $begingroup$

      There-and-back measurements still show the effect of an aether, because you can compare the results in different directions. For example, there-and-back along the direction of motion would show a different speed from there-and-back across it. This was the approach taken in the Michelson-Morley experiment.






      share|cite|improve this answer










      $endgroup$














      • $begingroup$
        Comments are not for extended discussion; this conversation has been moved to chat.
        $endgroup$
        – Chris
        May 19 at 16:54















      13
















      $begingroup$

      There-and-back measurements still show the effect of an aether, because you can compare the results in different directions. For example, there-and-back along the direction of motion would show a different speed from there-and-back across it. This was the approach taken in the Michelson-Morley experiment.






      share|cite|improve this answer










      $endgroup$














      • $begingroup$
        Comments are not for extended discussion; this conversation has been moved to chat.
        $endgroup$
        – Chris
        May 19 at 16:54













      13














      13










      13







      $begingroup$

      There-and-back measurements still show the effect of an aether, because you can compare the results in different directions. For example, there-and-back along the direction of motion would show a different speed from there-and-back across it. This was the approach taken in the Michelson-Morley experiment.






      share|cite|improve this answer










      $endgroup$



      There-and-back measurements still show the effect of an aether, because you can compare the results in different directions. For example, there-and-back along the direction of motion would show a different speed from there-and-back across it. This was the approach taken in the Michelson-Morley experiment.







      share|cite|improve this answer













      share|cite|improve this answer




      share|cite|improve this answer










      answered May 18 at 20:38









      Bob JacobsenBob Jacobsen

      8,47013 silver badges24 bronze badges




      8,47013 silver badges24 bronze badges














      • $begingroup$
        Comments are not for extended discussion; this conversation has been moved to chat.
        $endgroup$
        – Chris
        May 19 at 16:54
















      • $begingroup$
        Comments are not for extended discussion; this conversation has been moved to chat.
        $endgroup$
        – Chris
        May 19 at 16:54















      $begingroup$
      Comments are not for extended discussion; this conversation has been moved to chat.
      $endgroup$
      – Chris
      May 19 at 16:54




      $begingroup$
      Comments are not for extended discussion; this conversation has been moved to chat.
      $endgroup$
      – Chris
      May 19 at 16:54











      4
















      $begingroup$

      That the one-way speed of light is not observable plays a large role in keeping various relativistically-correct aether theories alive. Like general relativity, these new aether theories locally reduce to special relativity in the absence of nearby massive objects. Unlike general relativity, these new aether theories have a preferred universal frame of reference, typically a frame co-moving with the cosmic microwave background radiation.






      share|cite|improve this answer










      $endgroup$










      • 1




        $begingroup$
        If such theories cannot be ruled out, then why shouldn't they be amongst the respectable opinions that thinkers and researchers may choose to adopt and work with?
        $endgroup$
        – Steve
        May 19 at 0:40






      • 1




        $begingroup$
        @Steve “Choose to adopt and work with” is not a binary. People make different choices. Lots of people find GR useful; few find The Flying Spaghetti God hypothesis useful. Other theories are in between in popularity, i.e the MOND family. Nor is this decision time-invariant: theories and hypotheses are more and less popular as people show them more or less useful.
        $endgroup$
        – Bob Jacobsen
        May 19 at 1:32






      • 2




        $begingroup$
        @BobJacobsen, useful for what purpose? Perhaps when working with aether concepts attracts comparisons to the Flying Spaghetti God, you make it dis-useful for professional scientists to adopt such views and perform such work? And my point is to rebut the tone in which David speaks of apparently tenable theories being "kept alive" - the writer's implication being that these theories ought to be dead. Like I say, if they cannot be ruled out, then they are fair game for further scientific enquiry.
        $endgroup$
        – Steve
        May 19 at 1:53






      • 3




        $begingroup$
        @Steve, general relativity is a simple theory. Its only arbitrary parameter is Newton's gravitational constant. Other viable theories carry additional arbitrary parameters. In the limit that these parameters approach some value, typically zero, these theories become equivalent to general relativity. To date, all observations are consistent with these free parameters taking on the value that makes the alternative theory equivalent to general relativity.
        $endgroup$
        – David Hammen
        May 19 at 2:51






      • 1




        $begingroup$
        @DavidHammen, that's basically what I said above in my original comment to Bob's answer, that as it stands today (and as it stood in about 1920) Lorentz's aether theory (in its latter form) is equivalent to relativity - they are not mutually exclusive theories. It is only the general philosophical approach and concepts they employ which differ.
        $endgroup$
        – Steve
        May 19 at 9:13















      4
















      $begingroup$

      That the one-way speed of light is not observable plays a large role in keeping various relativistically-correct aether theories alive. Like general relativity, these new aether theories locally reduce to special relativity in the absence of nearby massive objects. Unlike general relativity, these new aether theories have a preferred universal frame of reference, typically a frame co-moving with the cosmic microwave background radiation.






      share|cite|improve this answer










      $endgroup$










      • 1




        $begingroup$
        If such theories cannot be ruled out, then why shouldn't they be amongst the respectable opinions that thinkers and researchers may choose to adopt and work with?
        $endgroup$
        – Steve
        May 19 at 0:40






      • 1




        $begingroup$
        @Steve “Choose to adopt and work with” is not a binary. People make different choices. Lots of people find GR useful; few find The Flying Spaghetti God hypothesis useful. Other theories are in between in popularity, i.e the MOND family. Nor is this decision time-invariant: theories and hypotheses are more and less popular as people show them more or less useful.
        $endgroup$
        – Bob Jacobsen
        May 19 at 1:32






      • 2




        $begingroup$
        @BobJacobsen, useful for what purpose? Perhaps when working with aether concepts attracts comparisons to the Flying Spaghetti God, you make it dis-useful for professional scientists to adopt such views and perform such work? And my point is to rebut the tone in which David speaks of apparently tenable theories being "kept alive" - the writer's implication being that these theories ought to be dead. Like I say, if they cannot be ruled out, then they are fair game for further scientific enquiry.
        $endgroup$
        – Steve
        May 19 at 1:53






      • 3




        $begingroup$
        @Steve, general relativity is a simple theory. Its only arbitrary parameter is Newton's gravitational constant. Other viable theories carry additional arbitrary parameters. In the limit that these parameters approach some value, typically zero, these theories become equivalent to general relativity. To date, all observations are consistent with these free parameters taking on the value that makes the alternative theory equivalent to general relativity.
        $endgroup$
        – David Hammen
        May 19 at 2:51






      • 1




        $begingroup$
        @DavidHammen, that's basically what I said above in my original comment to Bob's answer, that as it stands today (and as it stood in about 1920) Lorentz's aether theory (in its latter form) is equivalent to relativity - they are not mutually exclusive theories. It is only the general philosophical approach and concepts they employ which differ.
        $endgroup$
        – Steve
        May 19 at 9:13













      4














      4










      4







      $begingroup$

      That the one-way speed of light is not observable plays a large role in keeping various relativistically-correct aether theories alive. Like general relativity, these new aether theories locally reduce to special relativity in the absence of nearby massive objects. Unlike general relativity, these new aether theories have a preferred universal frame of reference, typically a frame co-moving with the cosmic microwave background radiation.






      share|cite|improve this answer










      $endgroup$



      That the one-way speed of light is not observable plays a large role in keeping various relativistically-correct aether theories alive. Like general relativity, these new aether theories locally reduce to special relativity in the absence of nearby massive objects. Unlike general relativity, these new aether theories have a preferred universal frame of reference, typically a frame co-moving with the cosmic microwave background radiation.







      share|cite|improve this answer













      share|cite|improve this answer




      share|cite|improve this answer










      answered May 19 at 0:06









      David HammenDavid Hammen

      34.8k7 gold badges60 silver badges114 bronze badges




      34.8k7 gold badges60 silver badges114 bronze badges










      • 1




        $begingroup$
        If such theories cannot be ruled out, then why shouldn't they be amongst the respectable opinions that thinkers and researchers may choose to adopt and work with?
        $endgroup$
        – Steve
        May 19 at 0:40






      • 1




        $begingroup$
        @Steve “Choose to adopt and work with” is not a binary. People make different choices. Lots of people find GR useful; few find The Flying Spaghetti God hypothesis useful. Other theories are in between in popularity, i.e the MOND family. Nor is this decision time-invariant: theories and hypotheses are more and less popular as people show them more or less useful.
        $endgroup$
        – Bob Jacobsen
        May 19 at 1:32






      • 2




        $begingroup$
        @BobJacobsen, useful for what purpose? Perhaps when working with aether concepts attracts comparisons to the Flying Spaghetti God, you make it dis-useful for professional scientists to adopt such views and perform such work? And my point is to rebut the tone in which David speaks of apparently tenable theories being "kept alive" - the writer's implication being that these theories ought to be dead. Like I say, if they cannot be ruled out, then they are fair game for further scientific enquiry.
        $endgroup$
        – Steve
        May 19 at 1:53






      • 3




        $begingroup$
        @Steve, general relativity is a simple theory. Its only arbitrary parameter is Newton's gravitational constant. Other viable theories carry additional arbitrary parameters. In the limit that these parameters approach some value, typically zero, these theories become equivalent to general relativity. To date, all observations are consistent with these free parameters taking on the value that makes the alternative theory equivalent to general relativity.
        $endgroup$
        – David Hammen
        May 19 at 2:51






      • 1




        $begingroup$
        @DavidHammen, that's basically what I said above in my original comment to Bob's answer, that as it stands today (and as it stood in about 1920) Lorentz's aether theory (in its latter form) is equivalent to relativity - they are not mutually exclusive theories. It is only the general philosophical approach and concepts they employ which differ.
        $endgroup$
        – Steve
        May 19 at 9:13












      • 1




        $begingroup$
        If such theories cannot be ruled out, then why shouldn't they be amongst the respectable opinions that thinkers and researchers may choose to adopt and work with?
        $endgroup$
        – Steve
        May 19 at 0:40






      • 1




        $begingroup$
        @Steve “Choose to adopt and work with” is not a binary. People make different choices. Lots of people find GR useful; few find The Flying Spaghetti God hypothesis useful. Other theories are in between in popularity, i.e the MOND family. Nor is this decision time-invariant: theories and hypotheses are more and less popular as people show them more or less useful.
        $endgroup$
        – Bob Jacobsen
        May 19 at 1:32






      • 2




        $begingroup$
        @BobJacobsen, useful for what purpose? Perhaps when working with aether concepts attracts comparisons to the Flying Spaghetti God, you make it dis-useful for professional scientists to adopt such views and perform such work? And my point is to rebut the tone in which David speaks of apparently tenable theories being "kept alive" - the writer's implication being that these theories ought to be dead. Like I say, if they cannot be ruled out, then they are fair game for further scientific enquiry.
        $endgroup$
        – Steve
        May 19 at 1:53






      • 3




        $begingroup$
        @Steve, general relativity is a simple theory. Its only arbitrary parameter is Newton's gravitational constant. Other viable theories carry additional arbitrary parameters. In the limit that these parameters approach some value, typically zero, these theories become equivalent to general relativity. To date, all observations are consistent with these free parameters taking on the value that makes the alternative theory equivalent to general relativity.
        $endgroup$
        – David Hammen
        May 19 at 2:51






      • 1




        $begingroup$
        @DavidHammen, that's basically what I said above in my original comment to Bob's answer, that as it stands today (and as it stood in about 1920) Lorentz's aether theory (in its latter form) is equivalent to relativity - they are not mutually exclusive theories. It is only the general philosophical approach and concepts they employ which differ.
        $endgroup$
        – Steve
        May 19 at 9:13







      1




      1




      $begingroup$
      If such theories cannot be ruled out, then why shouldn't they be amongst the respectable opinions that thinkers and researchers may choose to adopt and work with?
      $endgroup$
      – Steve
      May 19 at 0:40




      $begingroup$
      If such theories cannot be ruled out, then why shouldn't they be amongst the respectable opinions that thinkers and researchers may choose to adopt and work with?
      $endgroup$
      – Steve
      May 19 at 0:40




      1




      1




      $begingroup$
      @Steve “Choose to adopt and work with” is not a binary. People make different choices. Lots of people find GR useful; few find The Flying Spaghetti God hypothesis useful. Other theories are in between in popularity, i.e the MOND family. Nor is this decision time-invariant: theories and hypotheses are more and less popular as people show them more or less useful.
      $endgroup$
      – Bob Jacobsen
      May 19 at 1:32




      $begingroup$
      @Steve “Choose to adopt and work with” is not a binary. People make different choices. Lots of people find GR useful; few find The Flying Spaghetti God hypothesis useful. Other theories are in between in popularity, i.e the MOND family. Nor is this decision time-invariant: theories and hypotheses are more and less popular as people show them more or less useful.
      $endgroup$
      – Bob Jacobsen
      May 19 at 1:32




      2




      2




      $begingroup$
      @BobJacobsen, useful for what purpose? Perhaps when working with aether concepts attracts comparisons to the Flying Spaghetti God, you make it dis-useful for professional scientists to adopt such views and perform such work? And my point is to rebut the tone in which David speaks of apparently tenable theories being "kept alive" - the writer's implication being that these theories ought to be dead. Like I say, if they cannot be ruled out, then they are fair game for further scientific enquiry.
      $endgroup$
      – Steve
      May 19 at 1:53




      $begingroup$
      @BobJacobsen, useful for what purpose? Perhaps when working with aether concepts attracts comparisons to the Flying Spaghetti God, you make it dis-useful for professional scientists to adopt such views and perform such work? And my point is to rebut the tone in which David speaks of apparently tenable theories being "kept alive" - the writer's implication being that these theories ought to be dead. Like I say, if they cannot be ruled out, then they are fair game for further scientific enquiry.
      $endgroup$
      – Steve
      May 19 at 1:53




      3




      3




      $begingroup$
      @Steve, general relativity is a simple theory. Its only arbitrary parameter is Newton's gravitational constant. Other viable theories carry additional arbitrary parameters. In the limit that these parameters approach some value, typically zero, these theories become equivalent to general relativity. To date, all observations are consistent with these free parameters taking on the value that makes the alternative theory equivalent to general relativity.
      $endgroup$
      – David Hammen
      May 19 at 2:51




      $begingroup$
      @Steve, general relativity is a simple theory. Its only arbitrary parameter is Newton's gravitational constant. Other viable theories carry additional arbitrary parameters. In the limit that these parameters approach some value, typically zero, these theories become equivalent to general relativity. To date, all observations are consistent with these free parameters taking on the value that makes the alternative theory equivalent to general relativity.
      $endgroup$
      – David Hammen
      May 19 at 2:51




      1




      1




      $begingroup$
      @DavidHammen, that's basically what I said above in my original comment to Bob's answer, that as it stands today (and as it stood in about 1920) Lorentz's aether theory (in its latter form) is equivalent to relativity - they are not mutually exclusive theories. It is only the general philosophical approach and concepts they employ which differ.
      $endgroup$
      – Steve
      May 19 at 9:13




      $begingroup$
      @DavidHammen, that's basically what I said above in my original comment to Bob's answer, that as it stands today (and as it stood in about 1920) Lorentz's aether theory (in its latter form) is equivalent to relativity - they are not mutually exclusive theories. It is only the general philosophical approach and concepts they employ which differ.
      $endgroup$
      – Steve
      May 19 at 9:13











      0
















      $begingroup$

      The $x$ direction is not any different physically than the $-x$ direction. So light travelling towards $x-> infty$ should travel in the same way it travels towards $x-> -infty$ wether theres an aether or not.
      What should make a difference is if we measure the speed of light in the direction the body that emitted the light is moving through the aether and any other direction, for example, the direction perpendicular to this movement to see the difference a bit easier. However no difference in arrival times was measured.






      share|cite|improve this answer










      $endgroup$



















        0
















        $begingroup$

        The $x$ direction is not any different physically than the $-x$ direction. So light travelling towards $x-> infty$ should travel in the same way it travels towards $x-> -infty$ wether theres an aether or not.
        What should make a difference is if we measure the speed of light in the direction the body that emitted the light is moving through the aether and any other direction, for example, the direction perpendicular to this movement to see the difference a bit easier. However no difference in arrival times was measured.






        share|cite|improve this answer










        $endgroup$

















          0














          0










          0







          $begingroup$

          The $x$ direction is not any different physically than the $-x$ direction. So light travelling towards $x-> infty$ should travel in the same way it travels towards $x-> -infty$ wether theres an aether or not.
          What should make a difference is if we measure the speed of light in the direction the body that emitted the light is moving through the aether and any other direction, for example, the direction perpendicular to this movement to see the difference a bit easier. However no difference in arrival times was measured.






          share|cite|improve this answer










          $endgroup$



          The $x$ direction is not any different physically than the $-x$ direction. So light travelling towards $x-> infty$ should travel in the same way it travels towards $x-> -infty$ wether theres an aether or not.
          What should make a difference is if we measure the speed of light in the direction the body that emitted the light is moving through the aether and any other direction, for example, the direction perpendicular to this movement to see the difference a bit easier. However no difference in arrival times was measured.







          share|cite|improve this answer













          share|cite|improve this answer




          share|cite|improve this answer










          answered May 18 at 23:06









          Juan Pablo ArcilaJuan Pablo Arcila

          1278 bronze badges




          1278 bronze badges
























              0
















              $begingroup$

              This is not really intended to answer the question explicitly; rather to introduce a point in the traditional argument about a 'luminiferous aether', and more particularly the role of 'Michelson-Morley'-type experiments in repudiating the idea of such a medium for light propagation, which appears to be universally overlooked: namely, that since what was cast into considerable doubt, and effectively disproved by the original M-M experiment, was only the proposition that the Earth at all may move relatively to such an 'aether', one entirely plausible possibility nevertheless remains for such an observation--that nothing at all may be conceived to move relative to such an 'aether', in which case it may be concluded that the 'aether' is in fact unitary, universal and exclusive, and therefore equivalent to the fact of a unitary universal substance or fabric of which all of realty is comprised.



              Permitting this as a premise, what is immediately clear then is that since we too are composed of such a substance, it is not possible ever in any objective sense to ascertain its ultimate nature; and we are forever constrained solely to an understanding of its dynamic principles, an understanding to which we are admitted by virtue of the principle of resonance which must inhere within a universal unity: we are able continually to perceive the operation of its singular dynamic through this faculty of resonance defining consciousness itself.



              In the pursuit of such understanding however, we must first ascribe to such a unitary substance some correspondingly singular and exclusive property, 'a priori' or on the logical grounds of innate reason, which if it is chosen correctly represents the de facto equivalent of the substance itself, the knowledge of whose ultimate nature we are denied; and in this regard, we are perhaps indebted to ancient clarity--the single conceivable property of a universal unity is that it 'holds together' or coheres, according to what Aristotle and his forbears called a property of 'hexis' (compounded with 'entelos', effectively a 'direction', as of process), and which we are entitled since the days of Galileo to call a 'force', namely a 'cohesive force' not merely permeating reality but effectively constituting the entirety of it. No other means of apprehending that entirety or any other part of it from a given aspect exists except through the perception or inference of the action of such a force (and all the principal equations of physics further attest to this).



              Now since such a 'cohesive force' operating exclusively and throughout reality from every imaginable spatial point must further be imagined to operate from within space itself, thus from within a dimension of 'interior spatial depth' proceeding within any such locus--and no doubt M-theory and Kaluza-Klein theory before it alludes mathematically to such a dimension (noting that it is necessary in the conception of such a dimension only to extrapolate any 3rd dimension of Euclidean space interiorly provided that the properties of distribution of 'cohesive force' themselves are permitted to imply the multiplicity of 'space-time dimensions' in string theory)--, then it is necessary in order to account for the state of reality, that it does not for instance collapse upon itself at any such locus under the influence of this 'interior suction', not only that a perpetual disparity exists between the components of this 'cohesive force'--effectively constituting this dynamic universal substance (or re-imagined 'aether')--, but to infer a principle of equilibration between such loci laterally which amounts to its distribution, in linear components, over planar areas, and within volumes.



              It is this interplay, inevitably oscillatory, between the force operating 'into spatial depth' at any point, and its tendency to universal distribution laterally between loci under the impetus of an equilibrating principle implicit in a dynamic of cohesion or coherence which may be argued to imply not only the wave characteristics of the universal substance itself, of which the light wave is the primary and what may be imagined optimally regular manifestation, but a finite wave speed which implies both the finite speed of light and the basis of Einstein's second postulate of the invariance of c. The universal substance is therefore not to be construed as a medium for light propagation in the old-fashioned manner of an 'aether'; rather the light wave speed is the wave speed of that substance itself.



              Considering the basis of that wave dynamic intrinsic to what amounts to a 'unitary universal cohesive field' (comprising 'cohesive loci' defined by resultants or vectors), there is a limit to what may be said here; suffice to establish that an interaction between disparate components of 'cohesive force' or 'cohesive loci'* implies a resultant amounting to the elaboration of a further locus which is not only displaced in the direction of the stronger component, but because further interactions are implied between resultant loci, an oscillatory effect ensues with respect to an initial reference position whose progressive aspect due to that displacement further implies tendencies to recurrence of a given resultant with respect to that reference: viz. a wave effect is elaborated whose components or wave vectors are exclusively resultants in 'cohesive force'.



              However, since the fundamental wave dynamic and by inference light itself is more fundamentally a 'wave interference effect' between an interior spatial and effectively Euclidean spatial component, there arises not only a primary progressive effect implied by the disparity between these two components ostensible in the Euclidean spatial axis, but a further regularly recurring residual component implied with respect to an initial reference point in that axis by the properties of progression and recurrence of that primary dynamic. That is, in the case of the unitary wave principle imagined, that dynamic of recurrence is characterised by a residual component which is itself progressive and therefore definitive of a further level of resolution within this dynamic of wave recurrence, and it is this component which corresponds to and defines the wavelengths of EMR and in its condition of optimal regularity, the wavelengths of visible light. It is also this incrementally progressive aspect of the unitary wave interference principle in its various facets which endows the unitary cohesive space with its 'quantum nature' or properties. *(These linear resultants in 'cohesive force' may loosely be characterised as 'loci' by virtue of the predominance of the interior spatial component; moreover, all loci are effectively aspects of a singular universal context, thus of each other).



              With respect to the second postulate then, since within a singular substance characterised by the operation of a unitary 'cohesive force' in disparate components and by a universal wave principle, all motion may be understood eventually as wave motion, including that of a rigid body, the inertial fame moving with a velocity v and a light source attached to it in Einstein's scenario, and given that the elaboration of light--and the effect of our 'perceptual resonance' with it--is inherent within such a universal substance, then if it is supposed that within this singular medium light speed c represents the maximum ostensible wave speed intrinsic to that substance, as in the vacuum* (as one appreciates from Maxwell's equations), no manner of interference between or aggregation of wave velocities may exceed that speed c. The combination of a moving inertial frame, a light source and a light wave propagating from it constitutes a continuous wave effect with maximal wave speed. *(That specific maximum arises by virtue of the effect that wave progression implicit within such a medium in that limiting instance is innately imbued with a property of optimum regularity; corresponding to the principle of stationary action).



              Moreover, if it is further proposed that light itself derives more fundamentally in an interference effect between two wave components or vectors--or components of 'cohesive force'--, one projecting from the interior spatial dimension according to a particular geometry in which that dimension is represented in the pole of a cube (whose vertices correspond to interacting 'cohesive loci'), and one proceeding progressively in the Euclidean spatial dimension represented by the diagonal of the face of such a cube in which that first component projects, such that the familiar electromagnetic properties of light may be inferred from that interaction, then some conceptual latitude exists, effectively within the rotational aspect of such an interference effect in 'cubic cohesive space', for the accommodation of the velocity of a light source within the maximum allowable wave velocity of the universal medium.



              That is, at the origin of the light source--in whatever context this is imagined--, those components of a progressive recurrent wave dynamic constituting the velocity ascribed to its material substance imagined in the direction of its motion are effectively incorporated into the effect of propagating light itself in that direction, presumably in the form of its energy: viz. the velocity of a light source is effectively zero in the direction in which light wave speed is considered (for a light source moving at c must be indistinguishable from the propagating light itself). One may for example be required to imagine that a conservation of the L orbital angular momentum of electrons in the source as these yield kinetic energy to propagating photons implies an effective loss of linear velocity in the direction of motion of the source equivalent to that velocity of the source.



              I might add that since the discussion generally regarding the physical explanation for Einstein's second postulate appears ordinarily to conclude that no fundamental reason has yet been established for the evident veracity of this postulate or what is therefore effectively an axiom, one feels justified in indulging in creative speculation on the issue. Additionally, as one explores such a speculative geometric model of a unitary universal substance and its principle of universal resonance, explanations for certain otherwise inexplicable observations in physics, in QED in particular, become readily apparent; any and all of which are contingent upon the assumption of that unitary substance based on the premise of what is argued as a self-evident or axiomatic truth ('analytic a priori' after Kant): that of the inviolate universal unity of reality.






              share|cite|improve this answer










              $endgroup$



















                0
















                $begingroup$

                This is not really intended to answer the question explicitly; rather to introduce a point in the traditional argument about a 'luminiferous aether', and more particularly the role of 'Michelson-Morley'-type experiments in repudiating the idea of such a medium for light propagation, which appears to be universally overlooked: namely, that since what was cast into considerable doubt, and effectively disproved by the original M-M experiment, was only the proposition that the Earth at all may move relatively to such an 'aether', one entirely plausible possibility nevertheless remains for such an observation--that nothing at all may be conceived to move relative to such an 'aether', in which case it may be concluded that the 'aether' is in fact unitary, universal and exclusive, and therefore equivalent to the fact of a unitary universal substance or fabric of which all of realty is comprised.



                Permitting this as a premise, what is immediately clear then is that since we too are composed of such a substance, it is not possible ever in any objective sense to ascertain its ultimate nature; and we are forever constrained solely to an understanding of its dynamic principles, an understanding to which we are admitted by virtue of the principle of resonance which must inhere within a universal unity: we are able continually to perceive the operation of its singular dynamic through this faculty of resonance defining consciousness itself.



                In the pursuit of such understanding however, we must first ascribe to such a unitary substance some correspondingly singular and exclusive property, 'a priori' or on the logical grounds of innate reason, which if it is chosen correctly represents the de facto equivalent of the substance itself, the knowledge of whose ultimate nature we are denied; and in this regard, we are perhaps indebted to ancient clarity--the single conceivable property of a universal unity is that it 'holds together' or coheres, according to what Aristotle and his forbears called a property of 'hexis' (compounded with 'entelos', effectively a 'direction', as of process), and which we are entitled since the days of Galileo to call a 'force', namely a 'cohesive force' not merely permeating reality but effectively constituting the entirety of it. No other means of apprehending that entirety or any other part of it from a given aspect exists except through the perception or inference of the action of such a force (and all the principal equations of physics further attest to this).



                Now since such a 'cohesive force' operating exclusively and throughout reality from every imaginable spatial point must further be imagined to operate from within space itself, thus from within a dimension of 'interior spatial depth' proceeding within any such locus--and no doubt M-theory and Kaluza-Klein theory before it alludes mathematically to such a dimension (noting that it is necessary in the conception of such a dimension only to extrapolate any 3rd dimension of Euclidean space interiorly provided that the properties of distribution of 'cohesive force' themselves are permitted to imply the multiplicity of 'space-time dimensions' in string theory)--, then it is necessary in order to account for the state of reality, that it does not for instance collapse upon itself at any such locus under the influence of this 'interior suction', not only that a perpetual disparity exists between the components of this 'cohesive force'--effectively constituting this dynamic universal substance (or re-imagined 'aether')--, but to infer a principle of equilibration between such loci laterally which amounts to its distribution, in linear components, over planar areas, and within volumes.



                It is this interplay, inevitably oscillatory, between the force operating 'into spatial depth' at any point, and its tendency to universal distribution laterally between loci under the impetus of an equilibrating principle implicit in a dynamic of cohesion or coherence which may be argued to imply not only the wave characteristics of the universal substance itself, of which the light wave is the primary and what may be imagined optimally regular manifestation, but a finite wave speed which implies both the finite speed of light and the basis of Einstein's second postulate of the invariance of c. The universal substance is therefore not to be construed as a medium for light propagation in the old-fashioned manner of an 'aether'; rather the light wave speed is the wave speed of that substance itself.



                Considering the basis of that wave dynamic intrinsic to what amounts to a 'unitary universal cohesive field' (comprising 'cohesive loci' defined by resultants or vectors), there is a limit to what may be said here; suffice to establish that an interaction between disparate components of 'cohesive force' or 'cohesive loci'* implies a resultant amounting to the elaboration of a further locus which is not only displaced in the direction of the stronger component, but because further interactions are implied between resultant loci, an oscillatory effect ensues with respect to an initial reference position whose progressive aspect due to that displacement further implies tendencies to recurrence of a given resultant with respect to that reference: viz. a wave effect is elaborated whose components or wave vectors are exclusively resultants in 'cohesive force'.



                However, since the fundamental wave dynamic and by inference light itself is more fundamentally a 'wave interference effect' between an interior spatial and effectively Euclidean spatial component, there arises not only a primary progressive effect implied by the disparity between these two components ostensible in the Euclidean spatial axis, but a further regularly recurring residual component implied with respect to an initial reference point in that axis by the properties of progression and recurrence of that primary dynamic. That is, in the case of the unitary wave principle imagined, that dynamic of recurrence is characterised by a residual component which is itself progressive and therefore definitive of a further level of resolution within this dynamic of wave recurrence, and it is this component which corresponds to and defines the wavelengths of EMR and in its condition of optimal regularity, the wavelengths of visible light. It is also this incrementally progressive aspect of the unitary wave interference principle in its various facets which endows the unitary cohesive space with its 'quantum nature' or properties. *(These linear resultants in 'cohesive force' may loosely be characterised as 'loci' by virtue of the predominance of the interior spatial component; moreover, all loci are effectively aspects of a singular universal context, thus of each other).



                With respect to the second postulate then, since within a singular substance characterised by the operation of a unitary 'cohesive force' in disparate components and by a universal wave principle, all motion may be understood eventually as wave motion, including that of a rigid body, the inertial fame moving with a velocity v and a light source attached to it in Einstein's scenario, and given that the elaboration of light--and the effect of our 'perceptual resonance' with it--is inherent within such a universal substance, then if it is supposed that within this singular medium light speed c represents the maximum ostensible wave speed intrinsic to that substance, as in the vacuum* (as one appreciates from Maxwell's equations), no manner of interference between or aggregation of wave velocities may exceed that speed c. The combination of a moving inertial frame, a light source and a light wave propagating from it constitutes a continuous wave effect with maximal wave speed. *(That specific maximum arises by virtue of the effect that wave progression implicit within such a medium in that limiting instance is innately imbued with a property of optimum regularity; corresponding to the principle of stationary action).



                Moreover, if it is further proposed that light itself derives more fundamentally in an interference effect between two wave components or vectors--or components of 'cohesive force'--, one projecting from the interior spatial dimension according to a particular geometry in which that dimension is represented in the pole of a cube (whose vertices correspond to interacting 'cohesive loci'), and one proceeding progressively in the Euclidean spatial dimension represented by the diagonal of the face of such a cube in which that first component projects, such that the familiar electromagnetic properties of light may be inferred from that interaction, then some conceptual latitude exists, effectively within the rotational aspect of such an interference effect in 'cubic cohesive space', for the accommodation of the velocity of a light source within the maximum allowable wave velocity of the universal medium.



                That is, at the origin of the light source--in whatever context this is imagined--, those components of a progressive recurrent wave dynamic constituting the velocity ascribed to its material substance imagined in the direction of its motion are effectively incorporated into the effect of propagating light itself in that direction, presumably in the form of its energy: viz. the velocity of a light source is effectively zero in the direction in which light wave speed is considered (for a light source moving at c must be indistinguishable from the propagating light itself). One may for example be required to imagine that a conservation of the L orbital angular momentum of electrons in the source as these yield kinetic energy to propagating photons implies an effective loss of linear velocity in the direction of motion of the source equivalent to that velocity of the source.



                I might add that since the discussion generally regarding the physical explanation for Einstein's second postulate appears ordinarily to conclude that no fundamental reason has yet been established for the evident veracity of this postulate or what is therefore effectively an axiom, one feels justified in indulging in creative speculation on the issue. Additionally, as one explores such a speculative geometric model of a unitary universal substance and its principle of universal resonance, explanations for certain otherwise inexplicable observations in physics, in QED in particular, become readily apparent; any and all of which are contingent upon the assumption of that unitary substance based on the premise of what is argued as a self-evident or axiomatic truth ('analytic a priori' after Kant): that of the inviolate universal unity of reality.






                share|cite|improve this answer










                $endgroup$

















                  0














                  0










                  0







                  $begingroup$

                  This is not really intended to answer the question explicitly; rather to introduce a point in the traditional argument about a 'luminiferous aether', and more particularly the role of 'Michelson-Morley'-type experiments in repudiating the idea of such a medium for light propagation, which appears to be universally overlooked: namely, that since what was cast into considerable doubt, and effectively disproved by the original M-M experiment, was only the proposition that the Earth at all may move relatively to such an 'aether', one entirely plausible possibility nevertheless remains for such an observation--that nothing at all may be conceived to move relative to such an 'aether', in which case it may be concluded that the 'aether' is in fact unitary, universal and exclusive, and therefore equivalent to the fact of a unitary universal substance or fabric of which all of realty is comprised.



                  Permitting this as a premise, what is immediately clear then is that since we too are composed of such a substance, it is not possible ever in any objective sense to ascertain its ultimate nature; and we are forever constrained solely to an understanding of its dynamic principles, an understanding to which we are admitted by virtue of the principle of resonance which must inhere within a universal unity: we are able continually to perceive the operation of its singular dynamic through this faculty of resonance defining consciousness itself.



                  In the pursuit of such understanding however, we must first ascribe to such a unitary substance some correspondingly singular and exclusive property, 'a priori' or on the logical grounds of innate reason, which if it is chosen correctly represents the de facto equivalent of the substance itself, the knowledge of whose ultimate nature we are denied; and in this regard, we are perhaps indebted to ancient clarity--the single conceivable property of a universal unity is that it 'holds together' or coheres, according to what Aristotle and his forbears called a property of 'hexis' (compounded with 'entelos', effectively a 'direction', as of process), and which we are entitled since the days of Galileo to call a 'force', namely a 'cohesive force' not merely permeating reality but effectively constituting the entirety of it. No other means of apprehending that entirety or any other part of it from a given aspect exists except through the perception or inference of the action of such a force (and all the principal equations of physics further attest to this).



                  Now since such a 'cohesive force' operating exclusively and throughout reality from every imaginable spatial point must further be imagined to operate from within space itself, thus from within a dimension of 'interior spatial depth' proceeding within any such locus--and no doubt M-theory and Kaluza-Klein theory before it alludes mathematically to such a dimension (noting that it is necessary in the conception of such a dimension only to extrapolate any 3rd dimension of Euclidean space interiorly provided that the properties of distribution of 'cohesive force' themselves are permitted to imply the multiplicity of 'space-time dimensions' in string theory)--, then it is necessary in order to account for the state of reality, that it does not for instance collapse upon itself at any such locus under the influence of this 'interior suction', not only that a perpetual disparity exists between the components of this 'cohesive force'--effectively constituting this dynamic universal substance (or re-imagined 'aether')--, but to infer a principle of equilibration between such loci laterally which amounts to its distribution, in linear components, over planar areas, and within volumes.



                  It is this interplay, inevitably oscillatory, between the force operating 'into spatial depth' at any point, and its tendency to universal distribution laterally between loci under the impetus of an equilibrating principle implicit in a dynamic of cohesion or coherence which may be argued to imply not only the wave characteristics of the universal substance itself, of which the light wave is the primary and what may be imagined optimally regular manifestation, but a finite wave speed which implies both the finite speed of light and the basis of Einstein's second postulate of the invariance of c. The universal substance is therefore not to be construed as a medium for light propagation in the old-fashioned manner of an 'aether'; rather the light wave speed is the wave speed of that substance itself.



                  Considering the basis of that wave dynamic intrinsic to what amounts to a 'unitary universal cohesive field' (comprising 'cohesive loci' defined by resultants or vectors), there is a limit to what may be said here; suffice to establish that an interaction between disparate components of 'cohesive force' or 'cohesive loci'* implies a resultant amounting to the elaboration of a further locus which is not only displaced in the direction of the stronger component, but because further interactions are implied between resultant loci, an oscillatory effect ensues with respect to an initial reference position whose progressive aspect due to that displacement further implies tendencies to recurrence of a given resultant with respect to that reference: viz. a wave effect is elaborated whose components or wave vectors are exclusively resultants in 'cohesive force'.



                  However, since the fundamental wave dynamic and by inference light itself is more fundamentally a 'wave interference effect' between an interior spatial and effectively Euclidean spatial component, there arises not only a primary progressive effect implied by the disparity between these two components ostensible in the Euclidean spatial axis, but a further regularly recurring residual component implied with respect to an initial reference point in that axis by the properties of progression and recurrence of that primary dynamic. That is, in the case of the unitary wave principle imagined, that dynamic of recurrence is characterised by a residual component which is itself progressive and therefore definitive of a further level of resolution within this dynamic of wave recurrence, and it is this component which corresponds to and defines the wavelengths of EMR and in its condition of optimal regularity, the wavelengths of visible light. It is also this incrementally progressive aspect of the unitary wave interference principle in its various facets which endows the unitary cohesive space with its 'quantum nature' or properties. *(These linear resultants in 'cohesive force' may loosely be characterised as 'loci' by virtue of the predominance of the interior spatial component; moreover, all loci are effectively aspects of a singular universal context, thus of each other).



                  With respect to the second postulate then, since within a singular substance characterised by the operation of a unitary 'cohesive force' in disparate components and by a universal wave principle, all motion may be understood eventually as wave motion, including that of a rigid body, the inertial fame moving with a velocity v and a light source attached to it in Einstein's scenario, and given that the elaboration of light--and the effect of our 'perceptual resonance' with it--is inherent within such a universal substance, then if it is supposed that within this singular medium light speed c represents the maximum ostensible wave speed intrinsic to that substance, as in the vacuum* (as one appreciates from Maxwell's equations), no manner of interference between or aggregation of wave velocities may exceed that speed c. The combination of a moving inertial frame, a light source and a light wave propagating from it constitutes a continuous wave effect with maximal wave speed. *(That specific maximum arises by virtue of the effect that wave progression implicit within such a medium in that limiting instance is innately imbued with a property of optimum regularity; corresponding to the principle of stationary action).



                  Moreover, if it is further proposed that light itself derives more fundamentally in an interference effect between two wave components or vectors--or components of 'cohesive force'--, one projecting from the interior spatial dimension according to a particular geometry in which that dimension is represented in the pole of a cube (whose vertices correspond to interacting 'cohesive loci'), and one proceeding progressively in the Euclidean spatial dimension represented by the diagonal of the face of such a cube in which that first component projects, such that the familiar electromagnetic properties of light may be inferred from that interaction, then some conceptual latitude exists, effectively within the rotational aspect of such an interference effect in 'cubic cohesive space', for the accommodation of the velocity of a light source within the maximum allowable wave velocity of the universal medium.



                  That is, at the origin of the light source--in whatever context this is imagined--, those components of a progressive recurrent wave dynamic constituting the velocity ascribed to its material substance imagined in the direction of its motion are effectively incorporated into the effect of propagating light itself in that direction, presumably in the form of its energy: viz. the velocity of a light source is effectively zero in the direction in which light wave speed is considered (for a light source moving at c must be indistinguishable from the propagating light itself). One may for example be required to imagine that a conservation of the L orbital angular momentum of electrons in the source as these yield kinetic energy to propagating photons implies an effective loss of linear velocity in the direction of motion of the source equivalent to that velocity of the source.



                  I might add that since the discussion generally regarding the physical explanation for Einstein's second postulate appears ordinarily to conclude that no fundamental reason has yet been established for the evident veracity of this postulate or what is therefore effectively an axiom, one feels justified in indulging in creative speculation on the issue. Additionally, as one explores such a speculative geometric model of a unitary universal substance and its principle of universal resonance, explanations for certain otherwise inexplicable observations in physics, in QED in particular, become readily apparent; any and all of which are contingent upon the assumption of that unitary substance based on the premise of what is argued as a self-evident or axiomatic truth ('analytic a priori' after Kant): that of the inviolate universal unity of reality.






                  share|cite|improve this answer










                  $endgroup$



                  This is not really intended to answer the question explicitly; rather to introduce a point in the traditional argument about a 'luminiferous aether', and more particularly the role of 'Michelson-Morley'-type experiments in repudiating the idea of such a medium for light propagation, which appears to be universally overlooked: namely, that since what was cast into considerable doubt, and effectively disproved by the original M-M experiment, was only the proposition that the Earth at all may move relatively to such an 'aether', one entirely plausible possibility nevertheless remains for such an observation--that nothing at all may be conceived to move relative to such an 'aether', in which case it may be concluded that the 'aether' is in fact unitary, universal and exclusive, and therefore equivalent to the fact of a unitary universal substance or fabric of which all of realty is comprised.



                  Permitting this as a premise, what is immediately clear then is that since we too are composed of such a substance, it is not possible ever in any objective sense to ascertain its ultimate nature; and we are forever constrained solely to an understanding of its dynamic principles, an understanding to which we are admitted by virtue of the principle of resonance which must inhere within a universal unity: we are able continually to perceive the operation of its singular dynamic through this faculty of resonance defining consciousness itself.



                  In the pursuit of such understanding however, we must first ascribe to such a unitary substance some correspondingly singular and exclusive property, 'a priori' or on the logical grounds of innate reason, which if it is chosen correctly represents the de facto equivalent of the substance itself, the knowledge of whose ultimate nature we are denied; and in this regard, we are perhaps indebted to ancient clarity--the single conceivable property of a universal unity is that it 'holds together' or coheres, according to what Aristotle and his forbears called a property of 'hexis' (compounded with 'entelos', effectively a 'direction', as of process), and which we are entitled since the days of Galileo to call a 'force', namely a 'cohesive force' not merely permeating reality but effectively constituting the entirety of it. No other means of apprehending that entirety or any other part of it from a given aspect exists except through the perception or inference of the action of such a force (and all the principal equations of physics further attest to this).



                  Now since such a 'cohesive force' operating exclusively and throughout reality from every imaginable spatial point must further be imagined to operate from within space itself, thus from within a dimension of 'interior spatial depth' proceeding within any such locus--and no doubt M-theory and Kaluza-Klein theory before it alludes mathematically to such a dimension (noting that it is necessary in the conception of such a dimension only to extrapolate any 3rd dimension of Euclidean space interiorly provided that the properties of distribution of 'cohesive force' themselves are permitted to imply the multiplicity of 'space-time dimensions' in string theory)--, then it is necessary in order to account for the state of reality, that it does not for instance collapse upon itself at any such locus under the influence of this 'interior suction', not only that a perpetual disparity exists between the components of this 'cohesive force'--effectively constituting this dynamic universal substance (or re-imagined 'aether')--, but to infer a principle of equilibration between such loci laterally which amounts to its distribution, in linear components, over planar areas, and within volumes.



                  It is this interplay, inevitably oscillatory, between the force operating 'into spatial depth' at any point, and its tendency to universal distribution laterally between loci under the impetus of an equilibrating principle implicit in a dynamic of cohesion or coherence which may be argued to imply not only the wave characteristics of the universal substance itself, of which the light wave is the primary and what may be imagined optimally regular manifestation, but a finite wave speed which implies both the finite speed of light and the basis of Einstein's second postulate of the invariance of c. The universal substance is therefore not to be construed as a medium for light propagation in the old-fashioned manner of an 'aether'; rather the light wave speed is the wave speed of that substance itself.



                  Considering the basis of that wave dynamic intrinsic to what amounts to a 'unitary universal cohesive field' (comprising 'cohesive loci' defined by resultants or vectors), there is a limit to what may be said here; suffice to establish that an interaction between disparate components of 'cohesive force' or 'cohesive loci'* implies a resultant amounting to the elaboration of a further locus which is not only displaced in the direction of the stronger component, but because further interactions are implied between resultant loci, an oscillatory effect ensues with respect to an initial reference position whose progressive aspect due to that displacement further implies tendencies to recurrence of a given resultant with respect to that reference: viz. a wave effect is elaborated whose components or wave vectors are exclusively resultants in 'cohesive force'.



                  However, since the fundamental wave dynamic and by inference light itself is more fundamentally a 'wave interference effect' between an interior spatial and effectively Euclidean spatial component, there arises not only a primary progressive effect implied by the disparity between these two components ostensible in the Euclidean spatial axis, but a further regularly recurring residual component implied with respect to an initial reference point in that axis by the properties of progression and recurrence of that primary dynamic. That is, in the case of the unitary wave principle imagined, that dynamic of recurrence is characterised by a residual component which is itself progressive and therefore definitive of a further level of resolution within this dynamic of wave recurrence, and it is this component which corresponds to and defines the wavelengths of EMR and in its condition of optimal regularity, the wavelengths of visible light. It is also this incrementally progressive aspect of the unitary wave interference principle in its various facets which endows the unitary cohesive space with its 'quantum nature' or properties. *(These linear resultants in 'cohesive force' may loosely be characterised as 'loci' by virtue of the predominance of the interior spatial component; moreover, all loci are effectively aspects of a singular universal context, thus of each other).



                  With respect to the second postulate then, since within a singular substance characterised by the operation of a unitary 'cohesive force' in disparate components and by a universal wave principle, all motion may be understood eventually as wave motion, including that of a rigid body, the inertial fame moving with a velocity v and a light source attached to it in Einstein's scenario, and given that the elaboration of light--and the effect of our 'perceptual resonance' with it--is inherent within such a universal substance, then if it is supposed that within this singular medium light speed c represents the maximum ostensible wave speed intrinsic to that substance, as in the vacuum* (as one appreciates from Maxwell's equations), no manner of interference between or aggregation of wave velocities may exceed that speed c. The combination of a moving inertial frame, a light source and a light wave propagating from it constitutes a continuous wave effect with maximal wave speed. *(That specific maximum arises by virtue of the effect that wave progression implicit within such a medium in that limiting instance is innately imbued with a property of optimum regularity; corresponding to the principle of stationary action).



                  Moreover, if it is further proposed that light itself derives more fundamentally in an interference effect between two wave components or vectors--or components of 'cohesive force'--, one projecting from the interior spatial dimension according to a particular geometry in which that dimension is represented in the pole of a cube (whose vertices correspond to interacting 'cohesive loci'), and one proceeding progressively in the Euclidean spatial dimension represented by the diagonal of the face of such a cube in which that first component projects, such that the familiar electromagnetic properties of light may be inferred from that interaction, then some conceptual latitude exists, effectively within the rotational aspect of such an interference effect in 'cubic cohesive space', for the accommodation of the velocity of a light source within the maximum allowable wave velocity of the universal medium.



                  That is, at the origin of the light source--in whatever context this is imagined--, those components of a progressive recurrent wave dynamic constituting the velocity ascribed to its material substance imagined in the direction of its motion are effectively incorporated into the effect of propagating light itself in that direction, presumably in the form of its energy: viz. the velocity of a light source is effectively zero in the direction in which light wave speed is considered (for a light source moving at c must be indistinguishable from the propagating light itself). One may for example be required to imagine that a conservation of the L orbital angular momentum of electrons in the source as these yield kinetic energy to propagating photons implies an effective loss of linear velocity in the direction of motion of the source equivalent to that velocity of the source.



                  I might add that since the discussion generally regarding the physical explanation for Einstein's second postulate appears ordinarily to conclude that no fundamental reason has yet been established for the evident veracity of this postulate or what is therefore effectively an axiom, one feels justified in indulging in creative speculation on the issue. Additionally, as one explores such a speculative geometric model of a unitary universal substance and its principle of universal resonance, explanations for certain otherwise inexplicable observations in physics, in QED in particular, become readily apparent; any and all of which are contingent upon the assumption of that unitary substance based on the premise of what is argued as a self-evident or axiomatic truth ('analytic a priori' after Kant): that of the inviolate universal unity of reality.







                  share|cite|improve this answer













                  share|cite|improve this answer




                  share|cite|improve this answer










                  answered May 24 at 11:00









                  jeremiahjeremiah

                  297 bronze badges




                  297 bronze badges
























                      -1
















                      $begingroup$

                      Round - trip measurement of speed of light says nothing about the one - way speed of light. Effects of Lorentz contraction of moving bodies (MM experiment) and dilation of moving clock (Kennedy Thorndike experiment) had been first introduced in the framework of Lorentz aether theory.



                      Since the same mathematical formalism occurs in both, it is not possible to distinguish between Lorentz aether theory and special relativity by experiment.



                      The one-way speed of light is apparently anisotropic, the Sagnac effect (page 42 in this paper) is the best evidence of that.



                      Remember, that in rotating frames, even in special relativity, the non-transitivity of Einstein synchronization diminishes its usefulness. If clock 1 and clock 2 are not synchronized directly, but by using a chain of intermediate clocks, the synchronization depends on the path chosen. Synchronization around the circumference of a rotating disk gives a non-vanishing time difference that depends on the direction used.



                      Hence, the one – way speed of light relatively to the Earth surface is anisotropic, and it is very unlikely that one way speed of light is c an all relatively moving inertial frames.



                      On the simplest example of the floating in a water ships this paper simulates all kinematic effects of special relativity (length contraction, time dilation, relativistic velocity addition, relativistic and transverse Doppler effects, Twin paradox, Bell‘s spaceship paradox, symmetry of observations). Things that may seem quirky and unusual take very simple shape, as soon as "absolute" time and medium had been introduced.



                      The chapter Transverse Doppler Effect makes clear, that „conditionally moving“ clock ticks slower (transverse redshift) than „conditionally resting“ one , while „conditionally resting“ tick faster (transverse blueshift) than the "moving one". It is clear, that different synchronization procedures (standard isotropic for "stationary") frame and (non-standard anisotropic for "moving" frame) is needed to obtain the same result conducting measurements with the pair of synchronized clocks.



                      The article on special relativity in Wikipedia teaches, that: "The Principle of Invariant Light Speed – "... light is always propagated in empty space with a definite velocity [speed] c which is independent of the state of motion of the emitting body" (from the preface).[p 1] That is, light in vacuum propagates with the speed c (a fixed constant, independent of direction) in at least one system of inertial coordinates (the "stationary system"), regardless of the state of motion of the light source."



                      Apparently, according to the article, there is at least one "special stationary frame" where speed of light is isotropic and some others, where the speed of light is anisotropic.






                      share|cite|improve this answer












                      $endgroup$










                      • 2




                        $begingroup$
                        The Sagnac effect is most emphatically not evidence for light speed anisotropy. The Sagnac effect is predicted by special relativity which assumes isotropy. To claim that it demonstrates anisotropy is only evidence that you do not know your facts enough to produce a credible answer! -1
                        $endgroup$
                        – Dale
                        May 19 at 10:49






                      • 1




                        $begingroup$
                        Especially for YOU I have posted the links to the Sagnac effect explanation in Gron's paper (it is written black on white there, that in rotating frame speed of light is anisotropic ) and the link to Einstein synchronization in Wikipedia. Could you please be so kind to clean your glasses and read it again? Or maybe you think that the Earth does not rotate? These views are out to date, I am afraid to say.
                        $endgroup$
                        – Albert
                        May 19 at 11:00







                      • 1




                        $begingroup$
                        That the speed of light is anisotropic in a rotating reference frame does not imply that it is evidence for light speed anisotropy. For something to be evidence of a hypothesis it must not only be predicted by a hypothesis but also it must be not predicted by the alternative hypothesis. The Sagnac effect is equally predicted both a non-isotropic and an isotropic hypothesis. Therefore it is not evidence of either
                        $endgroup$
                        – Dale
                        May 19 at 11:10







                      • 1




                        $begingroup$
                        Another minus one. The Sagnac effect is not evidence that the one-way speed of light is anisotropic in inertial frames. It is evidence that the one-way speed of light is anisotropic in rotating frames, which of course are not inertial. The Sagnac effect is fully consistent with special relativity. If this observed fact was inconsistent with special relativity, we wouldn't be teaching special relativity any more. It would instead be yet another idea tossed on the "oops, that turned out to be wrong and useless" pile of failed theories, just like the many failed theories of heat.
                        $endgroup$
                        – David Hammen
                        May 19 at 15:24







                      • 1




                        $begingroup$
                        @Dale, agreed, excuse me on that point. I think when I wrote I attributed Albert's apparent mis-paraphrasing of you, to your own actual words. As long as we are all now agreed that there is no conclusive evidence either way, and the one-way speed may or may not be constant, then that is the state of current science on the question. Special relativity is a theory that treats the one-way speed as constant (without establishing the fact by any experiment), other theories are tenable which do not treat the one-way speed as constant.
                        $endgroup$
                        – Steve
                        May 19 at 19:22
















                      -1
















                      $begingroup$

                      Round - trip measurement of speed of light says nothing about the one - way speed of light. Effects of Lorentz contraction of moving bodies (MM experiment) and dilation of moving clock (Kennedy Thorndike experiment) had been first introduced in the framework of Lorentz aether theory.



                      Since the same mathematical formalism occurs in both, it is not possible to distinguish between Lorentz aether theory and special relativity by experiment.



                      The one-way speed of light is apparently anisotropic, the Sagnac effect (page 42 in this paper) is the best evidence of that.



                      Remember, that in rotating frames, even in special relativity, the non-transitivity of Einstein synchronization diminishes its usefulness. If clock 1 and clock 2 are not synchronized directly, but by using a chain of intermediate clocks, the synchronization depends on the path chosen. Synchronization around the circumference of a rotating disk gives a non-vanishing time difference that depends on the direction used.



                      Hence, the one – way speed of light relatively to the Earth surface is anisotropic, and it is very unlikely that one way speed of light is c an all relatively moving inertial frames.



                      On the simplest example of the floating in a water ships this paper simulates all kinematic effects of special relativity (length contraction, time dilation, relativistic velocity addition, relativistic and transverse Doppler effects, Twin paradox, Bell‘s spaceship paradox, symmetry of observations). Things that may seem quirky and unusual take very simple shape, as soon as "absolute" time and medium had been introduced.



                      The chapter Transverse Doppler Effect makes clear, that „conditionally moving“ clock ticks slower (transverse redshift) than „conditionally resting“ one , while „conditionally resting“ tick faster (transverse blueshift) than the "moving one". It is clear, that different synchronization procedures (standard isotropic for "stationary") frame and (non-standard anisotropic for "moving" frame) is needed to obtain the same result conducting measurements with the pair of synchronized clocks.



                      The article on special relativity in Wikipedia teaches, that: "The Principle of Invariant Light Speed – "... light is always propagated in empty space with a definite velocity [speed] c which is independent of the state of motion of the emitting body" (from the preface).[p 1] That is, light in vacuum propagates with the speed c (a fixed constant, independent of direction) in at least one system of inertial coordinates (the "stationary system"), regardless of the state of motion of the light source."



                      Apparently, according to the article, there is at least one "special stationary frame" where speed of light is isotropic and some others, where the speed of light is anisotropic.






                      share|cite|improve this answer












                      $endgroup$










                      • 2




                        $begingroup$
                        The Sagnac effect is most emphatically not evidence for light speed anisotropy. The Sagnac effect is predicted by special relativity which assumes isotropy. To claim that it demonstrates anisotropy is only evidence that you do not know your facts enough to produce a credible answer! -1
                        $endgroup$
                        – Dale
                        May 19 at 10:49






                      • 1




                        $begingroup$
                        Especially for YOU I have posted the links to the Sagnac effect explanation in Gron's paper (it is written black on white there, that in rotating frame speed of light is anisotropic ) and the link to Einstein synchronization in Wikipedia. Could you please be so kind to clean your glasses and read it again? Or maybe you think that the Earth does not rotate? These views are out to date, I am afraid to say.
                        $endgroup$
                        – Albert
                        May 19 at 11:00







                      • 1




                        $begingroup$
                        That the speed of light is anisotropic in a rotating reference frame does not imply that it is evidence for light speed anisotropy. For something to be evidence of a hypothesis it must not only be predicted by a hypothesis but also it must be not predicted by the alternative hypothesis. The Sagnac effect is equally predicted both a non-isotropic and an isotropic hypothesis. Therefore it is not evidence of either
                        $endgroup$
                        – Dale
                        May 19 at 11:10







                      • 1




                        $begingroup$
                        Another minus one. The Sagnac effect is not evidence that the one-way speed of light is anisotropic in inertial frames. It is evidence that the one-way speed of light is anisotropic in rotating frames, which of course are not inertial. The Sagnac effect is fully consistent with special relativity. If this observed fact was inconsistent with special relativity, we wouldn't be teaching special relativity any more. It would instead be yet another idea tossed on the "oops, that turned out to be wrong and useless" pile of failed theories, just like the many failed theories of heat.
                        $endgroup$
                        – David Hammen
                        May 19 at 15:24







                      • 1




                        $begingroup$
                        @Dale, agreed, excuse me on that point. I think when I wrote I attributed Albert's apparent mis-paraphrasing of you, to your own actual words. As long as we are all now agreed that there is no conclusive evidence either way, and the one-way speed may or may not be constant, then that is the state of current science on the question. Special relativity is a theory that treats the one-way speed as constant (without establishing the fact by any experiment), other theories are tenable which do not treat the one-way speed as constant.
                        $endgroup$
                        – Steve
                        May 19 at 19:22














                      -1














                      -1










                      -1







                      $begingroup$

                      Round - trip measurement of speed of light says nothing about the one - way speed of light. Effects of Lorentz contraction of moving bodies (MM experiment) and dilation of moving clock (Kennedy Thorndike experiment) had been first introduced in the framework of Lorentz aether theory.



                      Since the same mathematical formalism occurs in both, it is not possible to distinguish between Lorentz aether theory and special relativity by experiment.



                      The one-way speed of light is apparently anisotropic, the Sagnac effect (page 42 in this paper) is the best evidence of that.



                      Remember, that in rotating frames, even in special relativity, the non-transitivity of Einstein synchronization diminishes its usefulness. If clock 1 and clock 2 are not synchronized directly, but by using a chain of intermediate clocks, the synchronization depends on the path chosen. Synchronization around the circumference of a rotating disk gives a non-vanishing time difference that depends on the direction used.



                      Hence, the one – way speed of light relatively to the Earth surface is anisotropic, and it is very unlikely that one way speed of light is c an all relatively moving inertial frames.



                      On the simplest example of the floating in a water ships this paper simulates all kinematic effects of special relativity (length contraction, time dilation, relativistic velocity addition, relativistic and transverse Doppler effects, Twin paradox, Bell‘s spaceship paradox, symmetry of observations). Things that may seem quirky and unusual take very simple shape, as soon as "absolute" time and medium had been introduced.



                      The chapter Transverse Doppler Effect makes clear, that „conditionally moving“ clock ticks slower (transverse redshift) than „conditionally resting“ one , while „conditionally resting“ tick faster (transverse blueshift) than the "moving one". It is clear, that different synchronization procedures (standard isotropic for "stationary") frame and (non-standard anisotropic for "moving" frame) is needed to obtain the same result conducting measurements with the pair of synchronized clocks.



                      The article on special relativity in Wikipedia teaches, that: "The Principle of Invariant Light Speed – "... light is always propagated in empty space with a definite velocity [speed] c which is independent of the state of motion of the emitting body" (from the preface).[p 1] That is, light in vacuum propagates with the speed c (a fixed constant, independent of direction) in at least one system of inertial coordinates (the "stationary system"), regardless of the state of motion of the light source."



                      Apparently, according to the article, there is at least one "special stationary frame" where speed of light is isotropic and some others, where the speed of light is anisotropic.






                      share|cite|improve this answer












                      $endgroup$



                      Round - trip measurement of speed of light says nothing about the one - way speed of light. Effects of Lorentz contraction of moving bodies (MM experiment) and dilation of moving clock (Kennedy Thorndike experiment) had been first introduced in the framework of Lorentz aether theory.



                      Since the same mathematical formalism occurs in both, it is not possible to distinguish between Lorentz aether theory and special relativity by experiment.



                      The one-way speed of light is apparently anisotropic, the Sagnac effect (page 42 in this paper) is the best evidence of that.



                      Remember, that in rotating frames, even in special relativity, the non-transitivity of Einstein synchronization diminishes its usefulness. If clock 1 and clock 2 are not synchronized directly, but by using a chain of intermediate clocks, the synchronization depends on the path chosen. Synchronization around the circumference of a rotating disk gives a non-vanishing time difference that depends on the direction used.



                      Hence, the one – way speed of light relatively to the Earth surface is anisotropic, and it is very unlikely that one way speed of light is c an all relatively moving inertial frames.



                      On the simplest example of the floating in a water ships this paper simulates all kinematic effects of special relativity (length contraction, time dilation, relativistic velocity addition, relativistic and transverse Doppler effects, Twin paradox, Bell‘s spaceship paradox, symmetry of observations). Things that may seem quirky and unusual take very simple shape, as soon as "absolute" time and medium had been introduced.



                      The chapter Transverse Doppler Effect makes clear, that „conditionally moving“ clock ticks slower (transverse redshift) than „conditionally resting“ one , while „conditionally resting“ tick faster (transverse blueshift) than the "moving one". It is clear, that different synchronization procedures (standard isotropic for "stationary") frame and (non-standard anisotropic for "moving" frame) is needed to obtain the same result conducting measurements with the pair of synchronized clocks.



                      The article on special relativity in Wikipedia teaches, that: "The Principle of Invariant Light Speed – "... light is always propagated in empty space with a definite velocity [speed] c which is independent of the state of motion of the emitting body" (from the preface).[p 1] That is, light in vacuum propagates with the speed c (a fixed constant, independent of direction) in at least one system of inertial coordinates (the "stationary system"), regardless of the state of motion of the light source."



                      Apparently, according to the article, there is at least one "special stationary frame" where speed of light is isotropic and some others, where the speed of light is anisotropic.







                      share|cite|improve this answer















                      share|cite|improve this answer




                      share|cite|improve this answer








                      edited May 19 at 19:17

























                      answered May 19 at 9:44









                      AlbertAlbert

                      1,0843 silver badges10 bronze badges




                      1,0843 silver badges10 bronze badges










                      • 2




                        $begingroup$
                        The Sagnac effect is most emphatically not evidence for light speed anisotropy. The Sagnac effect is predicted by special relativity which assumes isotropy. To claim that it demonstrates anisotropy is only evidence that you do not know your facts enough to produce a credible answer! -1
                        $endgroup$
                        – Dale
                        May 19 at 10:49






                      • 1




                        $begingroup$
                        Especially for YOU I have posted the links to the Sagnac effect explanation in Gron's paper (it is written black on white there, that in rotating frame speed of light is anisotropic ) and the link to Einstein synchronization in Wikipedia. Could you please be so kind to clean your glasses and read it again? Or maybe you think that the Earth does not rotate? These views are out to date, I am afraid to say.
                        $endgroup$
                        – Albert
                        May 19 at 11:00







                      • 1




                        $begingroup$
                        That the speed of light is anisotropic in a rotating reference frame does not imply that it is evidence for light speed anisotropy. For something to be evidence of a hypothesis it must not only be predicted by a hypothesis but also it must be not predicted by the alternative hypothesis. The Sagnac effect is equally predicted both a non-isotropic and an isotropic hypothesis. Therefore it is not evidence of either
                        $endgroup$
                        – Dale
                        May 19 at 11:10







                      • 1




                        $begingroup$
                        Another minus one. The Sagnac effect is not evidence that the one-way speed of light is anisotropic in inertial frames. It is evidence that the one-way speed of light is anisotropic in rotating frames, which of course are not inertial. The Sagnac effect is fully consistent with special relativity. If this observed fact was inconsistent with special relativity, we wouldn't be teaching special relativity any more. It would instead be yet another idea tossed on the "oops, that turned out to be wrong and useless" pile of failed theories, just like the many failed theories of heat.
                        $endgroup$
                        – David Hammen
                        May 19 at 15:24







                      • 1




                        $begingroup$
                        @Dale, agreed, excuse me on that point. I think when I wrote I attributed Albert's apparent mis-paraphrasing of you, to your own actual words. As long as we are all now agreed that there is no conclusive evidence either way, and the one-way speed may or may not be constant, then that is the state of current science on the question. Special relativity is a theory that treats the one-way speed as constant (without establishing the fact by any experiment), other theories are tenable which do not treat the one-way speed as constant.
                        $endgroup$
                        – Steve
                        May 19 at 19:22













                      • 2




                        $begingroup$
                        The Sagnac effect is most emphatically not evidence for light speed anisotropy. The Sagnac effect is predicted by special relativity which assumes isotropy. To claim that it demonstrates anisotropy is only evidence that you do not know your facts enough to produce a credible answer! -1
                        $endgroup$
                        – Dale
                        May 19 at 10:49






                      • 1




                        $begingroup$
                        Especially for YOU I have posted the links to the Sagnac effect explanation in Gron's paper (it is written black on white there, that in rotating frame speed of light is anisotropic ) and the link to Einstein synchronization in Wikipedia. Could you please be so kind to clean your glasses and read it again? Or maybe you think that the Earth does not rotate? These views are out to date, I am afraid to say.
                        $endgroup$
                        – Albert
                        May 19 at 11:00







                      • 1




                        $begingroup$
                        That the speed of light is anisotropic in a rotating reference frame does not imply that it is evidence for light speed anisotropy. For something to be evidence of a hypothesis it must not only be predicted by a hypothesis but also it must be not predicted by the alternative hypothesis. The Sagnac effect is equally predicted both a non-isotropic and an isotropic hypothesis. Therefore it is not evidence of either
                        $endgroup$
                        – Dale
                        May 19 at 11:10







                      • 1




                        $begingroup$
                        Another minus one. The Sagnac effect is not evidence that the one-way speed of light is anisotropic in inertial frames. It is evidence that the one-way speed of light is anisotropic in rotating frames, which of course are not inertial. The Sagnac effect is fully consistent with special relativity. If this observed fact was inconsistent with special relativity, we wouldn't be teaching special relativity any more. It would instead be yet another idea tossed on the "oops, that turned out to be wrong and useless" pile of failed theories, just like the many failed theories of heat.
                        $endgroup$
                        – David Hammen
                        May 19 at 15:24







                      • 1




                        $begingroup$
                        @Dale, agreed, excuse me on that point. I think when I wrote I attributed Albert's apparent mis-paraphrasing of you, to your own actual words. As long as we are all now agreed that there is no conclusive evidence either way, and the one-way speed may or may not be constant, then that is the state of current science on the question. Special relativity is a theory that treats the one-way speed as constant (without establishing the fact by any experiment), other theories are tenable which do not treat the one-way speed as constant.
                        $endgroup$
                        – Steve
                        May 19 at 19:22








                      2




                      2




                      $begingroup$
                      The Sagnac effect is most emphatically not evidence for light speed anisotropy. The Sagnac effect is predicted by special relativity which assumes isotropy. To claim that it demonstrates anisotropy is only evidence that you do not know your facts enough to produce a credible answer! -1
                      $endgroup$
                      – Dale
                      May 19 at 10:49




                      $begingroup$
                      The Sagnac effect is most emphatically not evidence for light speed anisotropy. The Sagnac effect is predicted by special relativity which assumes isotropy. To claim that it demonstrates anisotropy is only evidence that you do not know your facts enough to produce a credible answer! -1
                      $endgroup$
                      – Dale
                      May 19 at 10:49




                      1




                      1




                      $begingroup$
                      Especially for YOU I have posted the links to the Sagnac effect explanation in Gron's paper (it is written black on white there, that in rotating frame speed of light is anisotropic ) and the link to Einstein synchronization in Wikipedia. Could you please be so kind to clean your glasses and read it again? Or maybe you think that the Earth does not rotate? These views are out to date, I am afraid to say.
                      $endgroup$
                      – Albert
                      May 19 at 11:00





                      $begingroup$
                      Especially for YOU I have posted the links to the Sagnac effect explanation in Gron's paper (it is written black on white there, that in rotating frame speed of light is anisotropic ) and the link to Einstein synchronization in Wikipedia. Could you please be so kind to clean your glasses and read it again? Or maybe you think that the Earth does not rotate? These views are out to date, I am afraid to say.
                      $endgroup$
                      – Albert
                      May 19 at 11:00





                      1




                      1




                      $begingroup$
                      That the speed of light is anisotropic in a rotating reference frame does not imply that it is evidence for light speed anisotropy. For something to be evidence of a hypothesis it must not only be predicted by a hypothesis but also it must be not predicted by the alternative hypothesis. The Sagnac effect is equally predicted both a non-isotropic and an isotropic hypothesis. Therefore it is not evidence of either
                      $endgroup$
                      – Dale
                      May 19 at 11:10





                      $begingroup$
                      That the speed of light is anisotropic in a rotating reference frame does not imply that it is evidence for light speed anisotropy. For something to be evidence of a hypothesis it must not only be predicted by a hypothesis but also it must be not predicted by the alternative hypothesis. The Sagnac effect is equally predicted both a non-isotropic and an isotropic hypothesis. Therefore it is not evidence of either
                      $endgroup$
                      – Dale
                      May 19 at 11:10





                      1




                      1




                      $begingroup$
                      Another minus one. The Sagnac effect is not evidence that the one-way speed of light is anisotropic in inertial frames. It is evidence that the one-way speed of light is anisotropic in rotating frames, which of course are not inertial. The Sagnac effect is fully consistent with special relativity. If this observed fact was inconsistent with special relativity, we wouldn't be teaching special relativity any more. It would instead be yet another idea tossed on the "oops, that turned out to be wrong and useless" pile of failed theories, just like the many failed theories of heat.
                      $endgroup$
                      – David Hammen
                      May 19 at 15:24





                      $begingroup$
                      Another minus one. The Sagnac effect is not evidence that the one-way speed of light is anisotropic in inertial frames. It is evidence that the one-way speed of light is anisotropic in rotating frames, which of course are not inertial. The Sagnac effect is fully consistent with special relativity. If this observed fact was inconsistent with special relativity, we wouldn't be teaching special relativity any more. It would instead be yet another idea tossed on the "oops, that turned out to be wrong and useless" pile of failed theories, just like the many failed theories of heat.
                      $endgroup$
                      – David Hammen
                      May 19 at 15:24





                      1




                      1




                      $begingroup$
                      @Dale, agreed, excuse me on that point. I think when I wrote I attributed Albert's apparent mis-paraphrasing of you, to your own actual words. As long as we are all now agreed that there is no conclusive evidence either way, and the one-way speed may or may not be constant, then that is the state of current science on the question. Special relativity is a theory that treats the one-way speed as constant (without establishing the fact by any experiment), other theories are tenable which do not treat the one-way speed as constant.
                      $endgroup$
                      – Steve
                      May 19 at 19:22





                      $begingroup$
                      @Dale, agreed, excuse me on that point. I think when I wrote I attributed Albert's apparent mis-paraphrasing of you, to your own actual words. As long as we are all now agreed that there is no conclusive evidence either way, and the one-way speed may or may not be constant, then that is the state of current science on the question. Special relativity is a theory that treats the one-way speed as constant (without establishing the fact by any experiment), other theories are tenable which do not treat the one-way speed as constant.
                      $endgroup$
                      – Steve
                      May 19 at 19:22



















                      draft saved

                      draft discarded















































                      Thanks for contributing an answer to Physics Stack Exchange!


                      • Please be sure to answer the question. Provide details and share your research!

                      But avoid


                      • Asking for help, clarification, or responding to other answers.

                      • Making statements based on opinion; back them up with references or personal experience.

                      Use MathJax to format equations. MathJax reference.


                      To learn more, see our tips on writing great answers.




                      draft saved


                      draft discarded














                      StackExchange.ready(
                      function ()
                      StackExchange.openid.initPostLogin('.new-post-login', 'https%3a%2f%2fphysics.stackexchange.com%2fquestions%2f480938%2fdoes-the-fact-that-we-can-only-measure-the-two-way-speed-of-light-undermine-the%23new-answer', 'question_page');

                      );

                      Post as a guest















                      Required, but never shown





















































                      Required, but never shown














                      Required, but never shown












                      Required, but never shown







                      Required, but never shown

































                      Required, but never shown














                      Required, but never shown












                      Required, but never shown







                      Required, but never shown









                      Popular posts from this blog

                      Tamil (spriik) Luke uk diar | Nawigatjuun

                      Align equal signs while including text over equalitiesAMS align: left aligned text/math plus multicolumn alignmentMultiple alignmentsAligning equations in multiple placesNumbering and aligning an equation with multiple columnsHow to align one equation with another multline equationUsing \ in environments inside the begintabularxNumber equations and preserving alignment of equal signsHow can I align equations to the left and to the right?Double equation alignment problem within align enviromentAligned within align: Why are they right-aligned?

                      Where does the image of a data connector as a sharp metal spike originate from?Where does the concept of infected people turning into zombies only after death originate from?Where does the motif of a reanimated human head originate?Where did the notion that Dragons could speak originate?Where does the archetypal image of the 'Grey' alien come from?Where did the suffix '-Man' originate?Where does the notion of being injured or killed by an illusion originate?Where did the term “sophont” originate?Where does the trope of magic spells being driven by advanced technology originate from?Where did the term “the living impaired” originate?